Politics in Russia (Late 80s to Present) HWK : 10 1-page Max Answers

This HWK is due on Sunday night at 22:00 pm NYC time .

Save Time On Research and Writing
Hire a Pro to Write You a 100% Plagiarism-Free Paper.
Get My Paper

 

Each answer must be approx. 1 page long (12 Times New Roman Font) except when asked to answer briefly where only 1 paragraph is needed.

 

1st question includes my answer to give you a sense of what I am looking for.

Save Time On Research and Writing
Hire a Pro to Write You a 100% Plagiarism-Free Paper.
Get My Paper

 

Writting must be your own & you must reference the attachements (1-2 reference per question minimum). 

 

You must also own a Kindle & buy able to purchase the following books (kindle version is the cheapest): 

“Armageddon Averted”  S. Kotkin 10 USD 

“The Political Economy of Russia” Neil Robinson 12 USD 

  

The references are useful for the following questions. 

Q1: Sun p.499-513 , beissengerntlsmcollapse, aronsoviet collapse, Armageddon Averted (p.1-112)

Q2: Sun p.409-513 , mikemcfaul, Armageddon Averted p. 113-192 

Q3: Q2 Refs + The Political Economy of Russia (p.15-35) 

Q4:  Q2& Q3 refs+ HandlemanComradeCriminal, 

Q5: EvangelistaChechnya , Guiliano

Q6: Arageddon Averted p.193-220, T_rimington_return_to_putin’s_Russia

Q7: Q6 Refs 

Q8: The political Economy of Russia p.15-35

Q9&10: Q6-8 Refs

 

Do not contact me if you do not have an international relations degree/background. I am looking for  Masters level quality work.

Michael Yared

Uni: May2125

Politics in Russia

Spring 2013

\Please type your answers in 12 point font.

Be sure to answer each part of every question. You should cite the authors assigned this semester when relevant using the following format: (author’s name). Also, be creative and open in offering your opinion when relevant. These questions are designed to get you to think, not necessarily to look for a single, “correct” answer. Each question is worth 10 points.

______

1.) Briefly describe two of the causes of the collapse of the Soviet Union in 1991. Describe one proximate cause of the collapse, as well as one structural cause — i.e. a process or phenomenon that began years before 1991 and is connected to the Soviet system. (The causes you identify may combine both proximate and structural elements; if so, state that this is the case.) (The answer to this question may be relatively brief.)

As newly appointed general secretary in 1985, Gorbatchev set out to “bridge the gap between socialism’s ideals and it’s disappointing realities within the context of the superpower competition” S. Kotkin, Armageddon Averted.

As the soviet economy stagnated Gorbatchev decided to reallocate a rapidly shrinking budget & political resources from the military to the economy by enacting “Perestroika”. He went a step further by introducing “Glasnost” so as to increase transparency & reduce the authority of the state. Glasnost offered a substantial part of the population unprecedented access to commercial culture and the values of capitalism. The credibility of the regime started to weaken as “new perceptions contributed to a change in attitudes toward the regime and “a shift in values” ” L. Aaron, Everything you know about the Soviet Union Collapse in wrong.

This “ideology of reformism” lead to growing calls from the apparat to reign back the pace of liberalization. In response, Gorbatchev tried to democratize the party with competitive elections & reorganize the party secretariat: Gorbatchev “was exchanging a unitary structure for a federalized one” S. Kotkin, Armageddon Averted.”

M. Beissinger argues in his piece: Nationalism and the Collapse of Soviet Communism that Glasnost allowed for the emergence of “political nationalism across multiple contexts in the Soviet Union” which in turn lead to dispersion amongst the Communist Party as to how to best respond to it thereby further weakening the central control mechanism.

In summary, by disrupting the planned economy through “Perestroika” and discrediting the party’s ideology through “Glasnost”, Gorbatchev wary of an internal power struggle destroyed the party’s central control mechanism which in turn lead to the ultimate collapse of the Union.

2.) President Yeltsin’s decision to use force against the Russian parliament in 1993 was a fateful one. Explain the nature of the conflict between the executive and legislative branches of the new Russian state. What was it about? (hint: it was about more than one thing). Second, what were some consequences of Yeltsin’s decision to use force, in particular for his government’s ability to pass laws and make policy in the several year aftermath of his decision in 1993?

3.) Yeltsin’s inheritance of the Soviet system with its myriad problems influenced the initial path that Yeltsin’s government chose in constructing the new Russian state. Choose one of Yeltsin government’s early policies or reforms and describe how it was informed by the experience of the Soviet era. How did either: the need to definitively end certain aspects of the Soviet system; curtail the power of certain actors from the Soviet era; or the fear of returning to Soviet era shape Yeltsin’s policies. (For example, you may discuss the sequencing of political and economic reforms; some aspect of economic policy; his reform of state structures; relations with the ethnic republics, etc.) Relevant authors: Stephen Kotkin, Mike McFaul, Neil Robnison; Ron Suny.

4.) How did the effects of privatization reduce Yeltsin’s popularity? To answer this, think about how the economic and social effects of voucher privatization (Phase I of privatization) negatively affected certain categories of the population. Also, think about how certain categories of the population benefitted from privatization (both Phase I and Phase II) and how this reduced Yeltsin’s popularity.

5.) Briefly explain how the two wars in Chechnya weakened the Russian state. Provide at least two reasons. (You may answer this briefly).

6.) How did the accountability of regional leaders shift (1) from the Soviet era to the Y eltsin presidency, and then (2) from Yeltsin to Putin? What is the relationship between accountability and democracy?

(Hint: To answer this, identify who the relevant actors (or types of institutions) were in each period in both the regions and in Moscow.

7.) Elections are an important indicator of democracy. Describe how elections to the Duma have changed from the Yeltsin period in the 1990s, to the Putin era in the 2000s. Why have certain parties been able to survive into the present?

8.) Describe two chronic problems with Russia’s economy (briefly) and their consequences for Russian politics, broadly defined.

Politics means any political or social issue in the country. The chronic problem can be something that occurred during the 1990s, the 2000s, or it can span all of the post-Soviet years. You may rely on any of your assigned readings, including the chapters by Neil Robinson, Gerald Easter, and Richard Sakwa (and Stephen Kotkin). You are not required to describe a problem that we discussed in class, though you may, obviously.

9.) How has the relationship between oligarchs and the state changed from Yeltsin to Putin?

10) How has Putin strengthened the Russian state? How has he weakened it?

You may discuss various aspects of the state: e.g., the formal institutions of the state (i.e. parliament; federalism; the Constitution); the state’s relationship to various actors: minority ethnic groups; the economy/powerful economic actors; the meaning of the war in Chechnya; etc.

This question is broad and interpretative. You can be creative. But do not try to address subjects we have not yet covered (e.g. relationship of the state to the legal system; civil society). And try not to repeat your earlier answers to other questions.

What is the state? State institutions; the state provides welfare to the population; the military and security services.

Wednesday, J an uar y 16, 2013 Follow

Everything You Think You Know About the
Collapse of the Soviet Union Is Wrong
*And why it matters today in a new age of revolution.

BY LEON ARON | JULY/AUGUST 2011

Every revolution is a surprise. Still, the latest Russian Revolution must be counted among the greatest of surprises. In the

years leading up to 1991, virtually no Western expert, scholar, official, or politician foresaw the impending collapse of the

Soviet Union, and with it one-party dictatorship, the state-owned economy, and the Kremlin’s control over its domestic

and Eastern European empires. Neither, with one exception, did Soviet dissidents nor, judging by their memoirs, future

revolutionaries themselves. When Mikhail Gorbachev became general secretary of the Communist Party in March 1985,

none of his contemporaries anticipated a revolutionary crisis. Although there were disagreements over the size and depth

of the Soviet system’s problems, no one thought them to be life-threatening, at least not anytime soon.

Whence such strangely universal shortsightedness? The failure of Western experts to anticipate the Soviet Union’s

collapse may in part be attributed to a sort of historical revisionism — call it anti-anti-communism — that tended to

HOME DIR ECTOR Y CHANNELS BLOGS LATEST AR TICLES POSTS ABOUT FP MAGAZINE AR CHIVE SEAR CH Search FP LOG IN

https://www.cambeywest.com/subscribe/?p=frp&f=gate2&s=I1208sft

http://www.facebook.com/foreign.policy.magazine

http://www.foreignpolicy.com/node/feed

Home

http://www.foreignpolicy.com/revolution_road

http://www.foreignpolicy.com/articles/2011/06/20/everything_you_think_you_know_about_the_collapse_of_the_soviet_union_is_wrong

http://www.foreignpolicy.com/issues/187/contents/

Home

http://www.foreignpolicy.com/articles/2011/06/20/everything_you_think_you_know_about_the_collapse_of_the_soviet_union_is_wrong?print=yes&hidecomments=yes&page=full#

http://www.foreignpolicy.com/articles/2011/06/20/everything_you_think_you_know_about_the_collapse_of_the_soviet_union_is_wrong?print=yes&hidecomments=yes&page=full#

http://www.foreignpolicy.com/articles/2011/06/20/everything_you_think_you_know_about_the_collapse_of_the_soviet_union_is_wrong?print=yes&hidecomments=yes&page=full#

http://www.foreignpolicy.com/articles/2011/06/20/everything_you_think_you_know_about_the_collapse_of_the_soviet_union_is_wrong?print=yes&hidecomments=yes&page=full#

http://www.foreignpolicy.com/articles/2011/06/20/everything_you_think_you_know_about_the_collapse_of_the_soviet_union_is_wrong?print=yes&hidecomments=yes&page=full#

http://www.foreignpolicy.com/advertising/

http://www.foreignpolicy.com/issues/current

http://www.foreignpolicy.com/archive

http://id.foreignpolicy.com/identity/public/login/options?next_url=http://www.foreignpolicy.com/articles/2011/06/20/everything_you_think_you_know_about_the_collapse_of_the_soviet_union_is_wrong?print=yes&hidecomments=yes&page=full&previous_url=http://www.foreignpolicy.com/articles/2011/06/20/everything_you_think_you_know_about_the_collapse_of_the_soviet_union_is_wrong?print=yes&hidecomments=yes&page=full&KeepThis=true&TB_iframe=true&height=600&width=900&modal=true

exaggerate the Soviet regime’s stability and legitimacy. Yet others who could hardly be considered soft on communism

were just as puzzled by its demise. One of the architects of the U.S. strategy in the Cold War, George Kennan, wrote that,

in reviewing the entire “history of international affairs in the modern era,” he found it “hard to think of any event more

strange and startling, and at first glance inexplicable, than the sudden and total disintegration and disappearance … of the

great power known successively as the Russian Empire and then the Soviet Union.” Richard Pipes, perhaps the leading

American historian of Russia as well as an advisor to U.S. President Ronald Reagan, called the revolution “unexpected.” A

collection of essays about the Soviet Union’s demise in a special 1993 issue of the conservative National Interest magazine

was titled “The Strange Death of Soviet Communism.”

Were it easier to understand, this collective lapse in judgment could have been safely consigned to a mental file containing

other oddities and caprices of the social sciences, and then forgotten. Yet even today, at a 20-year remove, the assumption

that the Soviet Union would continue in its current state, or at most that it would eventually begin a long, drawn-out

decline, seems just as rational a conclusion.

Indeed, the Soviet Union in 1985 possessed much of the same natural and human resources that it had 10 years before.

Certainly, the standard of living was much lower than in most of Eastern Europe, let alone the West. Shortages, food

rationing, long lines in stores, and acute poverty were endemic. But the Soviet Union had known far greater calamities

and coped without sacrificing an iota of the state’s grip on society and economy, much less surrendering it.

Nor did any key parameter of economic performance prior to 1985 point to a rapidly advancing disaster. From 1981 to

1985 the growth of the country’s GDP, though slowing down compared with the 1960s and 1970s, averaged 1.9 percent a

year. The same lackadaisical but hardly catastrophic pattern continued through 1989. Budget deficits, which since the

French Revolution have been considered among the prominent portents of a coming revolutionary crisis, equaled less

than 2 percent of GDP in 1985. Although growing rapidly, the gap remained under 9 percent through 1989 — a size most

economists would find quite manageable.

The sharp drop in oil prices, from $66 a barrel in 1980 to $20 a barrel in 1986 (in 2000 prices) certainly was a heavy blow

to Soviet finances. Still, adjusted for inflation, oil was more expensive in the world markets in 1985 than in 1972, and only

one-third lower than throughout the 1970s. And at the same time, Soviet incomes increased more than 2 percent in 1985,

and inflation-adjusted wages continued to rise in the next five years through 1990 at an average of over 7 percent.

Yes, the stagnation was obvious and worrisome. But as Wesleyan University professor Peter Rutland has pointed out,

“Chronic ailments, after all, are not necessarily fatal.” Even the leading student of the revolution’s economic causes,

Anders Åslund, notes that from 1985 to 1987, the situation “was not at all dramatic.”

From the regime’s point of view, the political circumstances were even less troublesome. After 20 years of relentless

suppression of political opposition, virtually all the prominent dissidents had been imprisoned, exiled (as Andrei

Sakharov had been since 1980), forced to emigrate, or had died in camps and jails.

There did not seem to be any other signs of a pre-revolutionary crisis either, including the other traditionally assigned

cause of state failure — external pressure. On the contrary, the previous decade was correctly judged to amount “to the

realization of all major Soviet military and diplomatic desiderata,” as American historian and diplomat Stephen

Sestanovich has written. Of course, Afghanistan increasingly looked like a long war, but for a 5-million-strong Soviet

http://books.google.com/books?id=Q_xTyZUEqkYC&lpg=PA165&ots=JllBqvzBrg&dq=%22chronic%20ailments%2C%20after%20all%2C%20are%20not%20necessarily%20fatal%22&pg=PA166#v=onepage&q=%22chronic%20ailments%22&f=false

http://books.google.com/books?id=T-05AqadLgMC&lpg=PA47&ots=BrZUrzqRbr&dq=%22was%20not%20at%20all%20dramatic%22%20%22Anders%20%C3%85slund%22&pg=PA48#v=onepage&q=%22not%20at%20all%20dramatic%22&f=false

http://findarticles.com/p/articles/mi_m2751/is_n31/ai_13991682/pg_3/?tag=content;col1

military force the losses there were negligible. Indeed, though the enormous financial burden of maintaining an empire

was to become a major issue in the post-1987 debates, the cost of the Afghan war itself was hardly crushing: Estimated at

$4 billion to $5 billion in 1985, it was an insignificant portion of the Soviet GDP.

Nor was America the catalyzing force. The “Reagan Doctrine” of resisting and, if possible, reversing the Soviet Union’s

advances in the Third World did put considerable pressure on the perimeter of the empire, in places like Afghanistan,

Angola, Nicaragua, and Ethiopia. Yet Soviet difficulties there, too, were far from fatal.

As a precursor to a potentially very costly competition, Reagan’s proposed Strategic Defense Initiative indeed was crucial –

– but it was far from heralding a military defeat, given that the Kremlin knew very well that effective deployment of space-

based defenses was decades away. Similarly, though the 1980 peaceful anti-communist uprising of the Polish workers had

been a very disturbing development for Soviet leaders, underscoring the precariousness of their European empire, by 1985

Solidarity looked exhausted. The Soviet Union seemed to have adjusted to undertaking bloody “pacifications” in Eastern

Europe every 12 years — Hungary in 1956, Czechoslovakia in 1968, Poland in 1980 — without much regard for the world’s

opinion.

This, in other words, was a Soviet Union at the height of its global power and influence, both in its own view and in the

view of the rest of the world. “We tend to forget,” historian Adam Ulam would note later, “that in 1985, no

government of a major state appeared to be as firmly in power, its policies as clearly set in their course, as that of the

USSR.”

Certainly, there were plenty of structural reasons — economic, political, social — why the Soviet Union should have

collapsed as it did, yet they fail to explain fully how it happened when it happened. How, that is, between 1985 and 1989,

in the absence of sharply worsening economic, political, demographic, and other structural conditions, did the state and

its economic system suddenly begin to be seen as shameful, illegitimate, and intolerable by enough men and women to

become doomed?

LIKE VIRTUALLY ALL modern revolutions, the latest Russian one was started by a hesitant liberalization “from

above” — and its rationale extended well beyond the necessity to correct the economy or make the international

environment more benign. The core of Gorbachev’s enterprise was undeniably idealistic: He wanted to build a more moral

Soviet Union.

For though economic betterment was their banner, there is little doubt that Gorbachev and his supporters first set out to

right moral, rather than economic, wrongs. Most of what they said publicly in the early days of perestroika now seems no

more than an expression of their anguish over the spiritual decline and corrosive effects of the Stalinist past. It was the

beginning of a desperate search for answers to the big questions with which every great revolution starts: What is a good,

dignified life? What constitutes a just social and economic order? What is a decent and legitimate state? What should such

a state’s relationship with civil society be?

“A new moral atmosphere is taking shape in the country,” Gorbachev told the Central Committee at the January 1987

meeting where he declared glasnost — openness — and democratization to be the foundation of his perestroika, or

restructuring, of Soviet society. “A reappraisal of values and their creative rethinking is under way.” Later, recalling his

feeling that “we couldn’t go on like that any longer, and we had to change life radically, break away from the past

http://books.google.com/books?id=vJwC34GQnhEC&lpg=PA27&ots=IgOPPNiL5B&dq=%22Political%20will%20and%20personal%20belief%22&pg=PA87#v=onepage&q=%22firmly%20in%20power%22&f=false

malpractices,” he called it his “moral position.”

In a 1989 interview, the “godfather of glasnost,” Aleksandr Yakovlev, recalled that, returning to the Soviet Union in 1983

after 10 years as the ambassador to Canada, he felt the moment was at hand when people would declare, “Enough! We

cannot live like this any longer. Everything must be done in a new way. We must reconsider our concepts, our approaches,

our views of the past and our future.… There has come an understanding that it is simply impossible to live as we lived

before — intolerably, humiliatingly.”

To Gorbachev’s prime minister Nikolai Ryzhkov, the “moral [nravstennoe] state of the society” in 1985 was its “most

terrifying” feature:

[We] stole from ourselves, took and gave bribes, lied in the reports, in newspapers, from high podiums, wallowed

in our lies, hung medals on one another. And all of this — from top to bottom and from bottom to top.

Another member of Gorbachev’s very small original coterie of liberalizers, Foreign Minister Eduard Shevardnadze, was

just as pained by ubiquitous lawlessness and corruption. He recalls telling Gorbachev in the winter of 1984-1985:

“Everything is rotten. It has to be changed.”

Back in the 1950s, Gorbachev’s predecessor Nikita Khrushchev had seen firsthand how precarious was the edifice of the

house that Stalin built on terror and lies. But this fifth generation of Soviet leaders was more confident of the regime’s

resilience. Gorbachev and his group appeared to believe that what was right was also politically manageable.

Democratization, Gorbachev declared, was “not a slogan but the essence of perestroika.” Many years later he told

interviewers:

The Soviet model was defeated not only on the economic and social levels; it was defeated on a cultural level. Our

society, our people, the most educated, the most intellectual, rejected that model on the cultural level because it

does not respect the man, oppresses him spiritually and politically.

That reforms gave rise to a revolution by 1989 was due largely to another “idealistic” cause: Gorbachev’s deep and

personal aversion to violence and, hence, his stubborn refusal to resort to mass coercion when the scale and depth of

change began to outstrip his original intent. To deploy Stalinist repression even to “preserve the system” would have been

a betrayal of his deepest convictions. A witness recalls Gorbachev saying in the late 1980s, “We are told that we should

pound the fist on the table,” and then clenching his hand in an illustrative fist. “Generally speaking,” continued the

general secretary, “it could be done. But one does not feel like it.”

THE ROLE OF ideas and ideals in bringing about the Russian revolution comes into even sharper relief when we look at

what was happening outside the Kremlin. A leading Soviet journalist and later a passionate herald of glasnost, Aleksandr

Bovin, wrote in 1988 that the ideals of perestroika had “ripened” amid people’s increasing “irritation” at corruption,

brazen thievery, lies, and the obstacles in the way of honest work. Anticipations of “substantive changes were in the air,”

another witness recalled, and they forged an appreciable constituency for radical reforms. Indeed, the expectations that

greeted the coming to power of Gorbachev were so strong, and growing, that they shaped his actual policy. Suddenly,

ideas themselves became a material, structural factor in the unfolding revolution.

The credibility of official ideology, which in Yakovlev’s words, held the entire Soviet political and economic system

together “like hoops of steel,” was quickly weakening. New perceptions contributed to a change in attitudes toward the

regime and “a shift in values.” Gradually, the legitimacy of the political arrangements began to be questioned. In an

instance of Robert K. Merton’s immortal “Thomas theorem” — “If men define situations as real, they are real in their

consequence” — the actual deterioration of the Soviet economy became consequential only after and because of a

fundamental shift in how the regime’s performance was perceived and evaluated.

Writing to a Soviet magazine in 1987, a Russian reader called what he saw around him a “radical break [perelom] in

consciousness.” We know that he was right because Russia’s is the first great revolution whose course was charted in

public opinion polls almost from the beginning. Already at the end of 1989, the first representative national public opinion

survey found overwhelming support for competitive elections and the legalization of parties other than the Soviet

Communist Party — after four generations under a one-party dictatorship and with independent parties still illegal. By

mid-1990, more than half those surveyed in a Russian region agreed that “a healthy economy” was more likely if “the

government allows individuals to do as they wish.” Six months later, an all-Russia poll found 56 percent supporting a

rapid or gradual transition to a market economy. Another year passed, and the share of the pro-market respondents

increased to 64 percent.

Those who instilled this remarkable “break in consciousness” were no different from those who touched off the other

classic revolutions of modern times: writers, journalists, artists. As Alexis de Tocqueville observed, such men and women

“help to create that general awareness of dissatisfaction, that solidified public opinion, which … creates effective demand

for revolutionary change.” Suddenly, “the entire political education” of the nation becomes the “work of its men of letters.”

And so it was in Soviet Russia. The lines to newspaper kiosks — sometimes crowds around the block that formed at six in

the morning, with each daily run often sold out in two hours — and the skyrocketing subscriptions to the leading liberal

newspapers and magazines testify to the devastating power of the most celebrated essayists of glasnost, or in Samuel

Johnson’s phrase, the “teachers of truth”: the economist Nikolai Shmelyov; the political philosophers Igor Klyamkin and

Alexander Tsypko; brilliant essayists like Vasily Selyunin, Yuri Chernichenko, Igor Vinogradov, and Ales Adamovich; the

journalists Yegor Yakovlev, Len Karpinsky, Fedor Burlatsky, and at least two dozen more.

To them, a moral resurrection was essential. This meant not merely an overhaul of the Soviet political and economic

systems, not merely an upending of social norms, but a revolution on the individual level: a change in the personal

character of the Russian subject. As Mikhail Antonov declared in a seminal 1987 essay, “So What Is Happening to Us?” in

the magazine Oktyabr, the people had to be “saved” — not from external dangers but “most of all from themselves, from

the consequences of those demoralizing processes that kill the noblest human qualities.” Saved how? By making the

nascent liberalization fateful, irreversible — not Khrushchev’s short-lived “thaw,” but a climate change. And what would

guarantee this irreversibility? Above all, the appearance of a free man who would be “immune to the recurrences of

spiritual slavery.” The weekly magazine Ogoniok, a key publication of glasnost, wrote in February 1989 that only “man

incapable of being a police informer, of betraying, and of lies, no matter in whose or what name, can save us from the re-

emergence of a totalitarian state.”

The circuitous nature of this reasoning — to save the people one had to save perestroika, but perestroika could be saved

only if it was capable of changing man “from within” — did not seem to trouble anyone. Those who thought out loud about

these matters seemed to assume that the country’s salvation through perestroika and the extrication of its people from the

spiritual morass were tightly — perhaps, inextricably — interwoven, and left it at that. What mattered was reclaiming the

people to citizenship from “serfdom” and “slavery.” “Enough!” declared Boris Vasiliev, the author of a popular novella of

the period about World War II, which was made into an equally well-received film. “Enough lies, enough servility,

enough cowardice. Let’s remember, finally, that we are all citizens. Proud citizens of a proud nation!”

DELVING INTO THE causes of the French Revolution, de Tocqueville famously noted that regimes overthrown in

revolutions tend to be less repressive than the ones preceding them. Why? Because, de Tocqueville surmised, though

people “may suffer less,” their “sensibility is exacerbated.”

As usual, Tocqueville was onto something hugely important. From the Founding Fathers to the Jacobins and Bolsheviks,

revolutionaries have fought under essentially the same banner: advancement of human dignity. It is in the search for

dignity through liberty and citizenship that glasnost’s subversive sensibility lives — and will continue to live. Just as the

pages of Ogoniok and Moskovskie Novosti must take pride of place next to Boris Yeltsin on the tank as symbols of the

latest Russian revolution, so should Internet pages in Arabic stand as emblems of the present revolution next to the

images of rebellious multitudes in Cairo’s Tahrir Square, the Casbah plaza in Tunis, the streets of Benghazi, and the

blasted towns of Syria. Languages and political cultures aside, their messages and the feelings they inspired were

remarkably similar.

The fruit-seller Mohamed Bouazizi, whose self-immolation set off the Tunisian uprising that began the Arab Spring of

2011, did so “not because he was jobless,” a demonstrator in Tunis told an American reporter, but “because he … went

to talk to the [local authorities] responsible for his problem and he was beaten — it was about the government.” In

Benghazi, the Libyan revolt started with the crowd chanting, “The people want an end to corruption!” In Egypt, the

crowds were “all about the self-empowerment of a long-repressed people no longer willing to be afraid, no longer willing

to be deprived of their freedom, and no longer willing to be humiliated by their own leaders,” New York Times columnist

Thomas Friedman reported from Cairo this February. He could have been reporting from Moscow in 1991.

“Dignity Before Bread!” was the slogan of the Tunisian revolution. The Tunisian economy had grown between 2 and 8

percent a year in the two decades preceding the revolt. With high oil prices, Libya on the brink of uprising also enjoyed an

economic boom of sorts. Both are reminders that in the modern world, economic progress is not a substitute for the pride

and self-respect of citizenship. Unless we remember this well, we will continue to be surprised — by the “color revolutions”

in the post-Soviet world, the Arab Spring, and, sooner or later, an inevitable democratic upheaval in China — just as we

were in Soviet Russia. “The Almighty provided us with such a powerful sense of dignity that we cannot tolerate the denial

of our inalienable rights and freedoms, no matter what real or supposed benefits are provided by ‘stable’ authoritarian

regimes,” the president of Kyrgyzstan, Roza Otunbayeva, wrote this March. “It is the magic of people, young and old,

men and women of different religions and political beliefs, who come together in city squares and announce that enough

is enough.”

Of course, the magnificent moral impulse, the search for truth and goodness, is only a necessary but not a sufficient

condition for the successful remaking of a country. It may be enough to bring down the ancien regime, but not to

overcome, in one fell swoop, a deep-seated authoritarian national political culture. The roots of the democratic

institutions spawned by morally charged revolutions may prove too shallow to sustain a functioning democracy in a

http://slavic.princeton.edu/resources/video/2435/

http://www.gettyimages.com/detail/111657330/AFP

http://www.newyorker.com/reporting/2011/04/04/110404fa_fact_coll

http://www.washingtonpost.com/wp-dyn/content/article/2011/03/07/AR2011030703899.html

society with precious little tradition of grassroots self-organization and self-rule. This is something that is likely to prove a

huge obstacle to the carrying out of the promise of the Arab Spring — as it has proved in Russia. The Russian moral

renaissance was thwarted by the atomization and mistrust bred by 70 years of totalitarianism. And though Gorbachev and

Yeltsin dismantled an empire, the legacy of imperial thinking for millions of Russians has since made them receptive to

neo-authoritarian Putinism, with its propaganda leitmotifs of “hostile encirclement” and “Russia rising off its knees.”

Moreover, the enormous national tragedy (and national guilt) of Stalinism has never been fully explored and atoned for,

corrupting the entire moral enterprise, just as the glasnost troubadours so passionately warned.

Which is why today’s Russia appears once again to be inching toward another perestroika moment. Although the market

reforms of the 1990s and today’s oil prices have combined to produce historically unprecedented prosperity for millions,

the brazen corruption of the ruling elite, new-style censorship, and open disdain for public opinion have spawned

alienation and cynicism that are beginning to reach (if not indeed surpass) the level of the early 1980s.

One needs only to spend a few days in Moscow talking to the intelligentsia or, better yet, to take a quick look at the blogs

on LiveJournal (Zhivoy Zhurnal), Russia’s most popular Internet platform, or at the sites of the top independent and

opposition groups to see that the motto of the 1980s — “We cannot live like this any longer!” — is becoming an article of

faith again. The moral imperative of freedom is reasserting itself, and not just among the limited circles of pro-democracy

activists and intellectuals. This February, the Institute of Contemporary Development, a liberal think tank chaired by

President Dmitry Medvedev, published what looked like a platform for the 2012 Russian presidential election:

In the past Russia needed liberty to live [better]; it must now have it in order to survive.… The challenge of our

times is an overhaul of the system of values, the forging of new consciousness. We cannot build a new country

with the old thinking.… The best investment [the state can make in man] is Liberty and the Rule of Law. And

respect for man’s Dignity.

It was the same intellectual and moral quest for self-respect and pride that, beginning with a merciless moral scrutiny of

the country’s past and present, within a few short years hollowed out the mighty Soviet state, deprived it of legitimacy, and

turned it into a burned-out shell that crumbled in August 1991. The tale of this intellectual and moral journey is an

absolutely central story of the 20th century’s last great revolution.

Save big when you subscribe to FP.
Stephen Ferry/Liaison/Getty Images

Leon Aron is director of Russian studies at the American Enterprise Institute and

author of the forthcoming Roads to the Temple: Truth, Memory, Ideas, and Ideals

in the Making of the Russian Revolution, 1987-1991.

(125) SHOW COMMENTS LOGIN OR REGISTER REPORT ABUSE

http://www.foreignpolicy.com/node/848301?showcomments=yes

http://www.foreignpolicy.com/user/login?destination=articles/2011/06/20/everything_you_think_you_know_about_the_collapse_of_the_soviet_union_is_wrong?commentspace=true

http://www.foreignpolicy.com/user/register?destination=welcome?final=articles/2011/06/20/everything_you_think_you_know_about_the_collapse_of_the_soviet_union_is_wrong

http://www.foreignpolicy.com/contact?c=2&s=Abuse:http://www.foreignpolicy.com/node/848301

https://www.cambeywest.com/subscribe/?p=frp&f=paid&s=I101AEL

FOLLOW US ON TWITTER | VISIT US ON FACEBOOK | FOLLOW US ON RSS | SUBSCRIBE TO FOREIGN POLICY

ABOUT FP | MEET THE STAFF | FOREIGN EDITIONS | REPRINT PERMISSIONS | ADVERTISING | WRITERS’ GUIDELINES | PRESS ROOM | WORK AT FP

SERVICES:SUBSCRIPTION SERVICES | ACADEMIC PROGRAM | FP ARCHIVE | REPRINT PERMISSIONS | FP REPORTS AND MERCHANDISE | SPECIAL REPORTS | BUY BACK ISSUES

PRIVACY POLICY | DISCLAIMER | CONTACT US

11 DUPONT CIRCLE NW, SUITE 600 | WASHINGTON, DC 20036 | PHONE: 202-728-7300 | FAX: 202-728-7342

FOREIGN POLICY IS PUBLISHED BY THE FP GROUP, A DIVISION OF THE WASHINGTON POST COMPANY

ALL CONTENTS ©2013 THE FOREIGN POLICY GROUP, LLC. ALL RIGHTS RESERVED.

http://www.facebook.com/foreign.policy.magazine

http://www.foreignpolicy.com/node/33012

https://www.cambeywest.com/subscribe/?p=frp&f=paid&s=I101AFSUB

http://www.foreignpolicy.com/about_us

http://www.foreignpolicy.com/articles/meet_the_staff

http://www.foreignpolicy.com/advertising/mag_international.php

http://www.foreignpolicy.com/articles/reprint_permissions

http://www.foreignpolicy.com/advertising/

http://www.foreignpolicy.com/articles/writers_guidelines

http://www.foreignpolicy.com/articles/press_room

http://www.foreignpolicy.com/articles/employment

http://www.foreignpolicy.com/articles/subscription_services

https://www.cambeywest.com/subscribe/?p=frp&f=paid&s=I101HFEDU&r=edu

http://www.foreignpolicy.com/archive

http://www.foreignpolicy.com/articles/reprint_permissions_and_syndication

http://www.foreignpolicy.com/articles/reports_and_merchandise

http://www.foreignpolicy.com/articles/special_reports

http://www.foreignpolicy.com/articles/back_issues

http://www.foreignpolicy.com/articles/privacy_policy

http://www.foreignpolicy.com/articles/disclaimer

http://www.foreignpolicy.com/articles/contact_us

Home

Home

Contemporary European History
http://journals.cambridge.org/CEH

Additional services for Contemporary European History:

Email alerts: Click here
Subscriptions: Click here
Commercial reprints: Click here
Terms of use : Click here

Nationalism and the Collapse of Soviet Communism

MARK R. BEISSINGER

Contemporary European History / Volume 18 / Special Issue 03 / August 2009, pp 331 ­ 347
DOI: 10.1017/S0960777309005074, Published online: 06 July 2009

Link to this article: http://journals.cambridge.org/abstract_S0960777309005074

How to cite this article:
MARK R. BEISSINGER (2009). Nationalism and the Collapse of Soviet Communism. 
Contemporary European History, 18, pp 331­347 doi:10.1017/S0960777309005074

Request Permissions : Click here

Downloaded from http://journals.cambridge.org/CEH, IP address: 128.59.163.237 on 17 Jan 2013

Nationalism and the Collapse

of Soviet Communism

M A R K R . B E I S S I N G E R

Abstract
This article examines the role of nationalism in the collapse of communism in the late 1980s and
early 1990s, arguing that nationalism (both in its presence and its absence, and in the various
conflicts and disorders that it unleashed) played an important role in structuring the way in which
communism collapsed. Two institutions of international and cultural control in particular –
the Warsaw Pact and ethnofederalism – played key roles in determining which communist
regimes failed and which survived. The article argues that the collapse of communism was not
a series of isolated, individual national stories of resistance but a set of interrelated streams of
activity in which action in one context profoundly affected action in other contexts – part of a
larger tide of assertions of national sovereignty that swept through the Soviet empire during this
period.

That nationalism should be considered among the causes of the collapse of
communism is not a view shared by everyone. A number of works on the end
of communism in the Soviet Union have argued, for instance, that nationalism
played only a minor role in the process – that the main events took place within
official institutions in Moscow and had relatively little to do with society, or that
nationalism was a marginal motivation or influence on the actions of those involved
in key decision-making. Failed institutions and ideologies, an economy in decline,
the burden of military competition with the United States and instrumental goals
of self-enrichment among the nomenklatura instead loom large in these accounts.1

In many narratives of the end of communism, nationalism is portrayed merely as a
consequence of communism’s demise, as a phase after communism disintegrated –
not as an autonomous or contributing force within the process of collapse itself.

237 Corwin Hall, Department of Politics, Princeton University, Princeton, NJ 08544, USA;
mbeissin@Princeton.edu.

1 See, for instance, Jerry F. Hough, Democratization and Revolution in the USSR, 1985–1991 (Washington,
DC: Brookings Institution, 1997); Steven Solnick, Stealing the State: Control and Collapse in Soviet
Institutions (Cambridge, MA: Harvard University Press, 1998); Stephen Kotkin, Armageddon Averted:
The Soviet Collapse, 1970–2000 (Oxford: Oxford University Press, 2001)

Contemporary European History, 18, 3 (2009), pp. 331–347 C© 2009 Cambridge University Press
doi:10.1017/S0960777309005074 Printed in the United Kingdom

332 Contemporary European History

Such a story, however, leaves a number of critical issues unaddressed. For one
thing, it completely ignores the critical mobilisational dimension of politics during
the 1987–92 period. Within the Soviet Union enormous mobilisations involving
millions of people occurred during these years, with nationalist demands being
the most prominent among the banners under which people mobilised. Indeed,
in the Soviet case regime change and the break-up of the Soviet state were not
entirely separable phases in the unfolding events that brought about the end of
communism, but were rather more overlapping and interrelated than many analyses
portray them to be. In 1988 and 1989 institutional opening politicised nationalism
across multiple contexts in the Soviet Union. These conflicts in turn magnified
divisions within the Communist Party over how to deal with them, encouraged the
spread of contention to other groups, created enormous disorder within institutions
and eventually led to the splintering of the Soviet state into national pieces. This
was an outcome that seemed utterly unimaginable to the vast majority of Soviet
citizens (and even most Soviet dissidents) when glasnost began in late 1986. It was
the unintended result of Mikhail Gorbachev’s policies – one that was made possible
not just by the widening political space that glasnost afforded, but also by the social
forces that moved into that space and utilised it to reconfigure regime and state.
Agency and contingency, not just structural determination, were important elements
of communism’s demise. Moreover, where nationalist mobilisation was weak (as in
Central Asia), communist elites survived the end of the Soviet Union, even while
the Soviet state collapsed around them. Indeed, to say that communism ended in
these cases begs the question, ‘in what respects?’ None of the post-Soviet states were
entirely new. They were all fragments of pre-independence state authority, and the
extent to which governing elites and bureaucracies were reconfigured in the post-
communist period ultimately depended on the degree to which they were challenged
from below by society during the glasnost period, principally through nationalist
mobilisation.2

But the argument that nationalism was marginal to communism’s demise also
provides an inadequate answer to the question of why some communist regimes
(China, North Korea, Vietnam, Laos, Cambodia and Cuba) survived the 1987–92
period. Many of these communist regimes also experienced ideological crises and
failed economies, were moving decisively toward market reform or were facing the
threat of increased military competition with the United States in the late 1980s
and early 1990s. Their economies were just as irrational, their governments just as
repressive and their bureaucracies just as corrupt as those European and Eurasian
communist regimes that failed. Yet Asian and Latin American communist regimes
survived while European and Eurasian communist regimes did not. Of course,
the chief reason why Asian and Latin American communist regimes survived is
that they never initiated the kind of political liberalisation undertaken inside the
Soviet Union, unleashing political forces that eventually overwhelmed the state.

2 Keith Darden and Anna Grzymala-Busse, ‘The Great Divide: Literacy, Nationalism, and the
Communist Collapse’, World Politics 59, 1 (October 2006), 83–115.

Nationalism and the Collapse of Soviet Communism 333

But another important difference has been the ability of Asian and Latin American
communist regimes to harness the nationalism of dominant national groups as a core
legitimating force, enabling these communist regimes to stigmatise foreign influences,
to marginalise more easily the oppositional challenges they have confronted and to
maintain their legitimacy within key sectors of society.3

By contrast, within European and Eurasian communist regimes in the late 1980s
nationalism largely failed as a legitimating force for communist regimes and served
instead as a major source for delegitimation and opposition.4 Whereas Russian
nationalism was long considered the linchpin of Soviet power, sustaining the Soviet
regime since the 1930s and mobilising critical support within Soviet society for
Soviet political domination throughout eastern Europe and Eurasia,5 for the most
part Russian nationalism failed to come to the defence of either communism or
the Soviet empire in the late 1980s. Instead, many Russians joined in the attacks,
ironically coming to identify themselves as victims of Soviet ‘imperial’ domination
and declaring Russian sovereignty vis-à-vis the Soviet government. In this sense,
Soviet communism was brought down in part by what Roman Szporluk perceptively
termed the ‘de-Sovietisation of Russia’6 – that is, the growing dissociation of Russians
and of Russian national identity from a state with which they had been routinely
identified in the past.

But it was not only the weakening Russian identification with the Soviet state
and its imperial project that facilitated communism’s collapse. The struggle against
what were widely viewed as repressive alien regimes imposed from without by Soviet
power was also a central animus underlying the events of 1989–91, both within
the Soviet Union and among its east European satellites. Communism in Europe
and Eurasia was more than just tyrannical rule, an idiotic economic system and a
ritualised ideology. It was also an international and multinational hierarchy of such
polities established and managed by Moscow – an interrelated structure of control that
replicated patterns of politics, economics and social organisation across geopolitical
space. Within Soviet-dominated eastern Europe, calls for popular sovereignty could
not easily be disentangled from those for independence from Muscovite tutelage,

3 See Martin K. Dimitrov, ‘Why Communism Didn’t Collapse: Exploring Regime Resilience in China,
Vietnam, Laos, North Korea, and Cuba’, paper presented at a conference on ‘Why Communism
Didn’t Collapse: Understanding Regime Resilience in China, Vietnam, Laos, North Korea, and
Cuba’, Dartmouth College, Hanover, NH, 25–26 May 2007.

4 The major exception was Yugoslavia. Minority nationalisms obviously played a major delegitimating
role in the collapse of Yugoslav communism and in the unmaking of the Yugoslav state. But Serbian
commitment to maintaining Yugoslavia’s territorial integrity and to Serbian communists who peddled
such an undertaking remained considerably stronger than the commitment of Russians to maintaining
the territorial integrity of the USSR, accounting for the outbreak of ethnic civil war in Yugoslavia
persistence of communist control in Serbia (in the guise of the Socialist Party) over the decade of the
1990s. See Veljko Vujačić, ‘Historical Legacies, Nationalist Mobilization, and Political Outcomes in
Russia and Serbia: A Weberian View’, Theory and Society 25, 6 (December 1996), 763–801.

5 David Brandenberger, National Bolshevism: Stalinist Mass Culture and the Formation of Modern Russian
National Identity, 1931–1956 (Cambridge, MA: Harvard University Press, 2002).

6 See, in particular, Roman Szporluk, Russia, Ukraine, and the Break-up of the Soviet Union (Stanford, CA:
Hoover Institution Press, 2000).

334 Contemporary European History

since these regimes had largely been imposed and maintained through intervention
and externally imposed controls. Thus behind the desire in 1989 for freedom stood
the desire for national sovereignty. In this sense, 1989 in eastern Europe was not merely
a series of revolts against communism as a repressive political and social system; it was
also a series of national revolts against Soviet domination, and as such closely related
to the same revolt that, by autumn 1989, had already become widespread within
Soviet society itself.

Precisely because nationalism was an underlying factor in the demise of
communism, the process of collapse largely spread along the two institutional forms
that were used to structure multinational and international control: ethnofederalism
and the Warsaw Pact. Both of these institutions utilised faux forms of sovereignty
to mask centralised control, so that the collapse of communism revolved in
significant part around making genuine the bogus sovereignties of communist-style
ethnofederalism and the Warsaw Pact. With the exception of Albania (explicable as
a simple case of regional spillover effects, and in fact the last of the east European
communist regimes to collapse), the other nine communist regimes that collapsed
in the late 1980s and early 1990s were either members of the Warsaw Pact, were
under the strong political domination of the USSR (Mongolia) or like the USSR
were ethnofederal states (Yugoslavia). By contrast, the six Asian and Latin American
communist regimes that survived stood outside the system of Soviet institutional
control, had established themselves independently from Soviet power and did not
employ ethnofederalism as an institutional form for mediating relations with their
own internal minorities.

In what follows I develop three arguments related to the role of nationalism in
the collapse of communism.7 First, nationalism (both in its presence, in its absence
and in the various conflicts and disorders it unleashed) played an important role
in structuring the way in which the collapse of communism unfolded. Of course,
to argue that nationalism was an important factor in structuring the collapse of
communism should not be interpreted as saying that nationalism ‘caused’ the collapse
of communism. History involves complex causation, and we would be fools to
constrain a series of events as complex as the collapse of communism within the
confines of any single causal factor. But, as we shall see, we would also be foolish
to ignore the national dimension to communism’s demise, not only because it was
central to the dynamic by which this demise materialised, but also because we would
seriously misunderstand post-communist politics and societies without elucidating
its national dimension. Second, nationalist mobilisation during this period was not
a series of individual nationalist stories. Rather, it was a set of interrelated streams
of activity in which action in one context exercised a profound effect on action
in other contexts – what I have called the ‘tidal’ context of nationalism. Indeed,
neither the Soviet state nor east European communism would likely have collapsed

7 These arguments are drawn from or are elaborations on my own work on the Soviet collapse. See
Mark R. Beissinger, Nationalist Mobilisation and the Collapse of the Soviet State (Cambridge: Cambridge
University Press, 2002).

Nationalism and the Collapse of Soviet Communism 335

had these nationalist revolts occurred in isolation from one another, so that these
interconnections were critical to the production of the collapse itself. Third, while
clearly structured, acts of nationalist mobilisation did not simply reflect a pre-existing
logic of institutions, structures and identities. Rather, acts of mobilisation also played
independent roles in transforming institutions, structures and identities, so that while
the collapse of communism is often portrayed as a structurally overdetermined drama8

(some would even say that communism’s collapse was predetermined from its very
establishment), its manifestation depended on myriad acts of defiance and contention
whose outcomes themselves were hardly predetermined.

Nationalism’s extraordinary appeal under glasnost

Gorbachev’s policy of glasnost and the political liberalisation that it produced were
obviously the critical institutional conditions that allowed the collapse of communism
to occur. Without glasnost, the forces that most directly brought about the collapse
could never have materialised or been able to act. But despite the absolute importance
of the Gorbachev factor and the broader factors that led Gorbachev to choose this
path, we should also remember that the collapse of communism was in fact the
unintended result of Gorbachev’s policies, not its conscious goal, and that the collapse
occurred precisely because other social forces moved into the widening political space
that glasnost afforded. Gorbachev sought to reform communism both domestically
and internationally, not to dismantle it. As Gorbachev recalled about the early years
of perestroika, ‘We talked not about revolution, but about improving the system. Then
we believed in such a possibility.’9 Gorbachev’s disavowal of the Brezhnev doctrine
in late 1988 similarly was not aimed at dismantling socialism in eastern Europe or
undoing the division of Germany, but rather at remaking Soviet relations with its
allies while undoing the cold war division of Europe. Of course, there was a great
deal about Gorbachev that was naive. But communism collapsed not only because
of Gorbachev’s policies, but also because social forces (in some places but not others)
utilised the opportunities that Gorbachev’s policies produced in order to mobilise
oppositions, transform institutions and identities, and appropriate power.

There is an unfortunate tendency in the literature on the collapse of communism
to draw a sharp line between events within the Soviet Union and those in eastern
Europe. Scholars of the Soviet collapse tend not to speak about a single annus
mirabilis,10 but of a five-year intense and protracted period in which new revelations
filled the newspapers every day, a dizzying array of institutional changes were enacted
and dozens (at times hundreds) of protests were mounted daily – many of them
spectacular events. From this perspective, the east European revolutions were but one
set of episodes (though a very critical set) in the events that constituted communism’s

8 For a critique of the heavy determinism in the literature on the breakdown of communism, see Stathis
N. Kalyvas, ‘The Decay and Breakdown of Communist One-Party Systems’, Annual Review of Political
Science, 2 (1999), 323–43.

9 Mikhail Gorbachev, Zhizn’ i reformy, Vol. 1 (Moscow: Novosti, 1995), 203 (emphasis in original).
10 Michael Howard, ‘The Springtime of Nations’, Foreign Affairs, 69, 1 (1990), 17–32.

336 Contemporary European History

collapse. An accurate understanding of the collapse of communism needs to view its
Soviet and east European dimensions as interrelated rather than separate processes.
What stood beneath this interrelationship was the ability of oppositions to draw
analogies across a wide expanse of political and cultural space, due to subjection
to common modes of domination and a shared sense of alien rule. It is here that
nationalism played a critical role in providing a frame through which analogies across
cultural and political boundaries were drawn.

The issues that effectively mobilised populations within the Soviet Union
during these years revolved precisely around nationalism. To be sure, issues
of democratisation, labour unrest and consumer shortages, and environmental
justice constituted autonomous vectors of mobilisation, at times intersecting with
nationalism and at times diverging from it. But as my own study of thousands of protest
demonstrations throughout the Soviet Union during the glasnost period showed,
nationalism gained a particular force and appeal not enjoyed by these other streams
of contention. For example, not only were demonstrations that voiced nationalist
demands but not democratising demands almost three times more frequent than those
that voiced democratising demands but not nationalist demands, but demonstrations
that voiced nationalist demands and did not raise democratising demands mobilised
ten times more participants than those voicing democratising demands but not raising
nationalist demands. The patterns are quite striking. Moreover, demonstrations that
combined both democratising and nationalist demands mobilised five times more
participants than those voicing democratising demands but not raising nationalist
demands. In other words, the strongest pressures from society for democratisation
came precisely from those movements that also pulled on nationalist tropes, and
without nationalism to underpin them demands for liberalisation on their own had
relatively weak resonance within Soviet society. A similar but even more pronounced
difference occurred between mobilisation over nationalist demands and mobilisation
over economic demands – in spite of the enormous decline in living standards that
occurred during this period.11 In short, nationalism exercised an unusual force of
attraction within the Soviet society during these years that was unparalleled by any
other set of issues.

The deeper causes for this were rooted in Soviet history and in the institutional
crisis of the Soviet state. Significant grievances revolving around the brutality of
the Stalinist past and the struggle for historical truth played prominent roles in
motivating nationalist mobilisation during glasnost. The Brezhnev era, lasting from
the accession of Leonid Brezhnev to the post of general secretary of the Communist
Party in October 1964 until Gorbachev’s selection as general secretary in March
1985, bred a sclerotic political system, a declining economy, widespread corruption,
and a deepening malaise and cynicism within society – all of which contributed
to a growing identification of the Soviet ruling elite as an alien other, even among
many ordinary Russians. The Soviet state and communist regime were closely fused,
since the multinational state had been founded by the communist regime, and the

11 Beissinger, Nationalist Mobilization, 75–9.

Nationalism and the Collapse of Soviet Communism 337

regime sought to legitimate itself primarily as an internationalist revolution. Yet
beneath the veneer of formal equality the reality of Russian dominance persisted,
reinforced in particular during Stalin’s rule, when a once multi-ethnic political elite
tipped towards disproportionate Russian representation, and a discourse of cultural
and political stratification came to be embraced. As a result of the fusion of state
and regime, any political opening that led to challenges against the regime was also
bound to politicise issues of stateness,12 particularly for groups like the Balts, who had
been incorporated forcefully into the USSR as a result of the Molotov-Ribbentrop
Pact of 1939. And because the state had been the creation of the regime, widespread
separatist challenges also necessarily assumed the form of anti-regime activity and were
unambiguous challenges to communist rule. Thus, regime change and the break-up
of the Soviet state were not easily separable phases in the demise of communism, but
were interrelated and partially concurrent phenomena.

As number of scholars have pointed out, many of the everyday institutional
practices of the Soviet state in the nationalities sphere (the ethnofederal system,
the primordialised passport system of ethnic identification and its use as a source
of discrimination in everyday life, the promotion of minority cultures within the
framework of the socialist state, and official personnel policies that promoted cadres in
part on the basis of nationality) also reinforced ethnicity over other (specifically, class)
modes of identity.13 Class identities had, of course, provided the initial underpinning
for communist ideology. But as modernisation and upward mobility proceeded, the
class basis of communism receded and the ethnic dimension of everyday life grew
more prominent.

Still, until glasnost, secessionist sentiments remained very much on the margins of
Soviet society – even in regions like the Baltic, where Soviet rule had come to be
seen as an unalterable fact of life and ‘a permanent state of affairs’.14 When glasnost
first began in late 1986 and early 1987, it contained no strong nationalist component,
and as an Estonian sociologist later observed, ‘neither its chief architects nor the
broad public were prepared for the possible rise of national movements’.15 Glasnost
initially manifested itself almost entirely in the operation of official institutions –
in the press, movie theatres and government offices. But already by spring 1987
glasnost began to escape official control, as small groups of hippies, Crimean Tatars,
ecologists, Jewish refuseniks, Russian nationalists and Baltic dissidents tested the
boundaries of the permissible by taking politics to the street, engaging in small-scale
demonstrations. The new atmosphere of press freedom, growing factionalism within

12 Valerie Bunce, Subversive Institutions: The Design and the Destruction of Socialism and the State (Cambridge:
Cambridge University Press, 1999).

13 Philip G. Roeder, Red Sunset: The Failure of Soviet Politics (Princeton, NJ: Princeton University Press,
1993); Ronald Grigor Suny, The Revenge of the Past: Nationalism, Revolution, and the Collapse of the
Soviet Union (Stanford, CA: Stanford University Press, 1993); Rogers Brubaker, ‘Nationhood and the
National Question in the Soviet Union and Post-Soviet Eurasia: An Institutionalist Account’, Theory
and Society, 23 (1994), 47–78.

14 Andrejs Plakans, The Latvians: A Short History (Stanford, CA: Stanford University Press, 1995), p. 162.
15 K. S. Hallik, quoted in Pravda, 7 June 1989, 2.

338 Contemporary European History

the Politburo and toleration of small-scale protest encouraged deeper politicisation. In
the early years of glasnost nationalist mobilisation followed closely upon the heels of
institutional reform, with key periods of institutional reform precipitating thickenings
of nationalist activity: the October 1987 Central Committee Plenum; the Nineteenth
Party Conference in June 1988; the March 1989 elections; and meetings of the First
Congress of People’s Deputies in July 1989. But by spring and summer 1989, large-
scale nationalist demonstrations involving hundreds of thousands of participants had
spread across multiple republics and had become a relatively frequent affair. By this
time the effect of institutional constraints on nationalist action had largely faded, and
nationalist mobilisation had increasingly become its own autonomous progenitor of
events, influencing the character of political institutions instead of being contained
by them.

Just how rapidly this transformation occurred is one of the astounding features
of the collapse of communism. It was not until February 1988 – over a year after
the initiation of glasnost – that the first major eruptions of nationalism occurred
in the Soviet Union: the massive Armenian protests over Karabakh, involving up
to a million people in Yerevan alone. Over the following nineteen months – from
February 1988 to August 1989 – the USSR experienced a veritable explosion of
nationalist mobilisation in the Baltic, the Transcaucasus, Ukraine and Moldova. By the
end of 1988 and the beginning of 1989, the coherence of Soviet control over its own
territory had been compromised by the rise to dominance of nationalist movements
within the Baltic republics and the veritable loss of control by the Soviet state over
events in Armenia and Azerbaijan. The massive mobilisations in Tbilisi in April 1989
that incited violent suppression by the Soviet army and the political backlash that this
evoked not only had undermined completely communist control in that republic,
but also convinced many throughout the Soviet Union and in the Soviet government
itself to question the utility of the deployment of the army as a means for containing
nationalist revolt. By summer 1989 the tide of nationalist contention spread to the
point that the Soviet regime appeared highly unstable. Enormous demonstrations
(involving hundreds of thousands of people, and sometimes up to a million) racked
all the republics of the Baltic and Transcaucasus at the time, spreading as well to
Western Ukraine and Moldova. During summer 1989, multiple violent inter-ethnic
conflicts also broke out across the southern tier of the USSR: between Uzbeks
and Meskhetian Turks, Kazakhs and Lezgins, Abkhaz and Georgians, Armenians
and Azerbaijanis, and Kyrgyz and Tajiks. Massive miner strikes in eastern Ukraine,
western Siberia and northern Kazakhstan – though non-national in character –
reflected the spread of large-scale protest to the Russian community, as well as the
growing disaffection of Russians from the Soviet state.

This mounting domestic incoherence and instability of the Soviet state was an
important part of the political opportunity structure that presented itself to east
Europeans in autumn 1989. If Balts could get away with declaring sovereignty vis-
à-vis the Soviet state at the end of 1988 and early 1989, and up to a million of
them could hold hands across the Baltic in August 1989 in favour of independence
from the USSR, why should Poles and Czechs not be expected to press their own
claims for popular sovereignty against their repressive, Kremlin-controlled regimes?

Nationalism and the Collapse of Soviet Communism 339

Why should Russians be afforded a greater degree of press freedom than Bulgarians
or East Germans (particularly when Soviet newspapers were readily available for
purchase throughout eastern Europe)? And if the Soviet state could not contain mass
revolts within its own borders, why should its client states in eastern Europe be
expected to contain them, even if they had been able to rely on Soviet help (which
Gorbachev had privately indicated would not be forthcoming)? Until early 1989
the pace of political change inside the Soviet Union outstripped the pace of change
within the Soviet Union’s Eastern Bloc allies, so that the example of political change
within the Soviet Union emboldened political reformers throughout the communist
world (and not only in eastern Europe, as the Chinese example illustrates). By
early 1989 reform efforts were already under way in Poland and Hungary, leading
to free elections in Poland in June 1989 and to the opening of borders and the
transition to political pluralism in Hungary. This in turn led to a dizzying three-
month cascade of events in late 1989: massive demonstrations in East Germany,
the fall of the Berlin Wall, the Velvet Revolution in Czechoslovakia, unrest and the
removal of Zhivkov in Bulgaria, and the violent overthrow of the Ceauşescu regime in
Romania.

In turn, the collapse of communism in eastern Europe enormously accelerated and
radicalised processes of nationalist revolt within the Soviet Union itself, leading to a
sense that a momentum had built up against the Soviet state that could no longer be
contained. The Ukrainian nationalist movement Rukh, for instance, actively utilised
the east European example to mobilise support for its cause. ‘The peoples of Poland,
Hungary, East Germany, and Czechoslovakia have said no to communist dictatorship’,
its banners at a demonstration read. ‘The next word is ours, citizens!’16 The first half
of 1990 saw a sharp rise in the number of groups pressing separatist demands inside
the Soviet Union, spurred on in particular by republican elections, which brought to
power nationalist movements in many republics and led to a bifurcation of authority
(dvoevlastie) and increasingly bitter disputes over sovereignty. It was at this time as
well that Gorbachev’s popularity plummeted among Russians and nomenklatura
elites began to defect from the centre in significant numbers, reinventing themselves
as nationalists in anticipation that Soviet power would not last much longer. The
classic example was Leonid Kravchuk. A party propagandist who once had been an
implacable enemy of Rukh, Kravchuk came, in the course of 1990, to embrace the
cause of Ukrainian sovereignty and independence. That once loyal nomenklatura like
Kravchuk could reconfigure themselves as ‘father’ of their respective nations was not
a plausible outcome outside these cross-case influences, for there would be no reason
why, in isolation from what had occurred elsewhere, these elites would have ever
considered defection.

The transnationalism of nationalism

Thus nationalist mobilisation during the collapse of communism was not a collection
of separate stories, but a series of interrelated streams of activity in which action

16 Ekspress khronika, no. 51, 17 December 1989, 1.

340 Contemporary European History

in one context exercised a profound effect on action in other contexts – what I
have called elsewhere a ‘tide’ of nationalism. This tidal dimension is often lost in
the literature on the collapse that focuses on a single country or on national cases.
The interconnectedness produced by common targets of mobilisation, common
institutional characteristics, common ideologies and common modes of domination
meant that oppositions also perceived a linkage of political opportunities, feeding
the spread of contention across cultural and political boundaries. The upsurge of
mobilisation across multiple contexts was produced not by a single shock, but rather
by the way in which agents forged connections with the challenging actions of others
through analogy and emulation. Institutional arrangements like ethnofederalism
or the Warsaw Pact became lightning rods for the lateral spread of contention,
because they connected populations in analogous ways. When such analogies
cohered, the example of successful contention in one context weakened political
order in other contexts by raising expectations among challengers that authority could
be successfully challenged. Challengers looked towards each other for inspiration
and ideas, widely borrowing tactics, frames and even programmes from those who
demonstrated prior successes. Nationalism is often portrayed as parochial and inward-
looking, lacking empathy and incapable of identifying with others. But the collapse
of communism illustrates the limits of such stereotypes. Most nationalist movements
are actually transnational in orientation, forced by strategic circumstances to conceive
of their fates as intertwined with others.

But the transnational spread of nationalist mobilisation was more than just a matter
of analogy and emulation. Those movements that gained early success also consciously
sought to spread their contention laterally so as to increase overall chances of
consolidating their victories by gaining allies and by further disrupting the coherence
of the state they wished to undermine. After the Nineteenth Party Conference in June
1988, attempts to challenge the Soviet regime proliferated with great rapidity, diffusing
across multiple groups. At this very time challenging groups engaged in a widespread
sharing of information, pamphlets, expertise, modes of challenge and mobilisational
frames. By June 1988 representatives of Ukrainian, Armenian, Georgian, Latvian,
Lithuanian and Estonian dissident nationalist movements had initiated contact with
one another and established a coordinating committee among themselves. Indeed,
in summer and autumn 1988 popular fronts created along the lines of the Baltic
model sprang up throughout most of the Soviet Union. Representatives of these
groups met frequently, shared documents and ideas, and occasionally aided each
other by providing material support or organising demonstrations in solidarity with
each other’s demands. The tide of nationalism thus assumed concrete form during the
collapse of communism in the ways in which nationalist paradigms were consciously
exported and borrowed, organisational resources were shared and challenging groups
sought inspiration from one another.

Arguably the most important mobilisational frame to emerge within the Soviet
Union during the glasnost era was the anti-imperial sovereignty frame that played
such an important role in the ultimate demise of the Soviet state. In its final years
of existence the imperial persona implicit within the Soviet state came to be openly

Nationalism and the Collapse of Soviet Communism 341

affirmed, as nations claimed sovereignty up to and including their place on the political
map of the world. This anti-imperial sovereignty frame first gained mass resonance in
the Baltic in summer 1988 and subsequently spread massively to Georgia, Armenia,
Azerbaijan, Moldova, Ukraine and, eventually, to Russia itself. When Boris Yeltsin
embraced Russian sovereignty vis-à-vis the USSR in June 1990, he was borrowing
from the tide of nationalism that had already swept across much of the USSR (Or,
as one Politburo member put it, ‘To make Russia sovereign is the golden daydream
of the Balts.’)17 So successful was the spread of this sovereignty frame that over the
course of 1990 every Soviet republic (as well as autonomous republics and even
one island in the Far East) issued their own declaration of sovereignty vis-à-vis the
Soviet government in what came to be known as the ‘parade of sovereignties’. The
diffusion of this anti-imperial sovereignty frame beyond the Baltic was partly an
attempt to capitalise on the prior success of others – a process of emulation typical of
modular phenomena like nationalism. But it was more than this. Baltic popular fronts
consciously attempted to reproduce themselves throughout the Soviet Union, out of
both philosophical and strategic considerations. They vigorously organised to extend
their influence throughout the Soviet Union for aiding the spread of the master frame
they themselves had pioneered. A conscious strategy of spreading secessionist revolt
laterally was pursued, both as an effort to consolidate secessionist movements through
the power of numbers and to weaken the regime by undermining its ability to defuse
nationalist challenges.18

It is unlikely that the Soviet state or east European communism would have ever
collapsed had these revolts occurred in isolation from one another. Certainly, had
the Balts engaged in their struggle alone, there is little doubt that they would easily
have been repressed. By contrast, the fact that claims of sovereignty against the centre
had spread broadly throughout the fabric of Soviet society made rebellion difficult to
contain. Part of the dilemma that had confronted opponents of Soviet communism
throughout its history was that past east European revolts against Soviet control had
exerted only limited influence inside the Soviet Union19 and had been repeatedly cut
short by Soviet intervention and pressure. In 1989, however, extensive revolt inside
the Soviet Union was occurring at the same time as east Europeans pressed for their
own freedom, so that the Kremlin for the first time faced a situation of multiple,
simultaneous revolts both within and outside the country. The modular spread of
revolt across the Soviet Union and eastern Europe represented an unusual period
of heightened contention that transcended cultural and international borders and in
which challenges to the state multiplied and fed off one another, overwhelming the
capacity of the state to contain them and evoking large-scale tectonic change in the
character of the state system.

17 Vadim Medvedev, quoted in Soiuz mozhno bylo sokhranit’ (Moscow: Aprel’-85, 1995), 64.
18 Nils R. Muiznieks, ‘The Influence of the Baltic Popular Movements on the Process of Soviet

Disintegration’, Europe-Asia Studies, 47, 1 (1995), 3–25.
19 See Roman Szporluk, ed., The Influence of East Europe and the Soviet West on the USSR (New York:

Praeger, 1975).

342 Contemporary European History

The weakness of Russian defence of the Soviet state

Nationalism was conspicuous in the collapse of communism not only by its presence,
but also by its absence. By all measures of conventional wisdom, Russians should
have been expected to come to the defence of Soviet communism and the Soviet
empire. Soviet communism was widely viewed as Russian communism, and Leninist
ideology was said to have resonated powerfully with embedded elements of Russian
political culture.20 Indeed, one of the reasons why earlier waves of revolt against Soviet
control had failed was precisely the way in which Russians had come to the defence
of the realm. Yet, in the late 1980s, at a time when the Soviet state liberalised and
Russian dominance was under attack, this did not happen. Instead, large numbers
of Russians protested against the Soviet state and acquiesced in its forfeiture of
empire.

How does one explain the weakness of Russian imperial nationalism in the
context of glasnost? To be sure, glasnost itself is to a large extent responsible, for
its constant revelations of Soviet abuses and atrocities drove a wedge between many
ordinary Russians and the Soviet state. But part of the explanation is also to be
found in the multiple political roles that Russians could and did assume during
these years. Russians were the dominant nationality of the Soviet Union and had
the most to lose from attempts to undermine the Soviet empire. But Russians also
constituted a disproportionate share of the Soviet intelligentsia and working class
relative to most other nationalities.21 The former were strongly attracted to ideas of
liberalisation, while the latter (due to their vulnerable position at a time of growing
economic shortage and insecurity) were most likely to protest against the regime’s
economic policies. This split structural position in relation to the changes introduced
by perestroika in fact led to a trifurcation of Russian mobilisation into nationalist-
conservative, liberal and labour-economic streams, each of which comprehended its
relationship to the Soviet state in different terms.

In this respect Russian mobilisation differed substantially from that of other groups
in the USSR, for it was unusually divided. Not only was there a plethora of Russian
movements by 1988–89, but these movements stood for quite distinct, and in some
instances opposing, frames. Rather than generating a nationalist backlash among
Russians, as many observers had expected, the tide of nationalism instead drove a
wedge more deeply between Russians, politicising and polarising cleavages among
them. Russian liberals eventually forged an alliance with non-Russian separatists
against the Soviet regime, borrowing their sovereignty and anti-colonial frames. They
did not define themselves as nationalists. They saw themselves as struggling primarily
against the communist regime, not for the nation. Yet in the first half of 1990 they
adopted many of the tropes of national liberation then extant elsewhere in the USSR,
coming to advocate a brand of liberal nationalism in which Russian sovereignty

20 Nikolai Berdyaev, The Origins of Russian Communism (Ann Arbor, MI: University of Michigan Press,
1960).

21 Darrell Slider, ‘A Note on the Class Structure of Soviet Nationalities’, Soviet Studies, 37, 4 (October
1985), pp. 535–540.

Nationalism and the Collapse of Soviet Communism 343

and self-determination were seen as necessary parts of the democratisation process.
In 1990 and 1991 this defence of Russian sovereignty against an overbearing and
imperial all-union government became the dominant theme of Russian mobilisation.
Similarly, as the economy deteriorated and the Soviet state disintegrated, labour
activism radicalised, coming in many cases to embrace the dismantlement of central
planning and the sovereignty paradigm.

Conservative-nationalists, by contrast, remained highly divided. Some distanced
themselves from the communist regime; indeed, demands for Russian sovereignty
initially emerged not from liberals, but from Russian nationalists, who, seeking
to counter the ‘Russophobia’ prevalent at the time, noted that Russians also
had been discriminated against and victimised by communism. Others embraced
a conservative communism that emphasised the defence of the party and the
state. But nationalist-conservatives failed to find a mass base for themselves within
Russia. Their attempts to court the coalminers of Ukraine, Siberia and northern
Kazakhstan also came to nought. Their support proved to be greatest within the
Russian-speaking communities of the Baltic and Moldova, but even here much of
their capacity to mobilise opposition to nationalist movements weakened in 1990
and 1991, as the break-up of the Soviet state grew imminent. In short, Russian
nationalism fizzled out as a force for defending the Soviet empire because glasnost
significantly undermined Russian support for the communist regime, Russians
were deeply divided politically and Russians increasingly embraced the sovereignty
paradigm championed by nationalist oppositions under the influence of the tide of
nationalism.

Structure and agency within ‘thickened history’

While the events in eastern Europe in 1989 are widely referred to as revolutions,
with the exception of the Baltic states it is not fashionable today to talk about the
collapse of the Soviet Union in these same terms. After all, in some Soviet republics
political power ultimately remained in the hands of communist officials, while in other
republics nationalist revolts descended into intra-ethnic violence and even civil war.
But the disintegration of the Soviet Union unambiguously deserves to be understood
as revolutionary. It easily falls within Tilly’s minimalist, processual understanding of
revolution (a situation of dual sovereignty in which non-ruling contenders mobilise
large numbers of citizens for the purpose of gaining control over the state).22 Even if
we assume a more robust, outcome-oriented definition such as that used by Skocpol
(the rapid transformation of a country’s state and class structures and its dominant
ideology),23 there is little doubt that the collapse of communism was revolutionary. In
most (though not all) Soviet republics, property relations were totally reconfigured in
the wake of communism’s collapse, longstanding social institutions were dismantled,

22 Charles Tilly, European Revolutions, 1492–1992 (Oxford: Blackwell, 1993).
23 Theda Skocpol, States and Social Revolutions: A Comparative Analysis of France, Russia, and China

(Cambridge University Press, 1979).

344 Contemporary European History

new ideologies and new classes came to the fore and new forms of social behaviour
sprang into existence.

The disintegration of the Soviet Union was accompanied by immense
transformations in political discourse and in public perceptions of politics. A
population that could barely imagine the break-up of their country came, within
a compressed period of time, to view its disintegration as inevitable. The record of
public opinion polling during these years demonstrates massive transformations in
attitudes toward the Soviet state – even for groups like the Balts, among whom the
notion of independence, once considered the pipe-dream of dissidents, came to be
almost unanimously embraced under the shifting boundaries of the possible. In the
case of the Ukrainians the transformation in attitudes under the influence of external
events was stark – to the point that 90 per cent of the Ukrainian population voted
in December 1991 for independent statehood in a national referendum. As Bohdan
Nahaylo described it,

What appears to have happened is that swiftly and almost imperceptibly . . . a revolution occurred
in the minds of Ukraine’s inhabitants. Somehow, during a remarkably short period, the idea of
Ukrainian independence, for so long depicted in the Soviet press as the hopeless cause of diehard
nationalists in Western Ukraine, took hold throughout the republic.24

Even large numbers of Russians, under the influence of events elsewhere, by
December 1991 came to support the dissolution of the USSR, as public opinion polls
at the time showed (although nostalgia for the USSR quickly developed thereafter).25

This enormous transformation in outlooks was of course facilitated by specific
structural conditions: the institutional and ideological crisis of the Soviet state, the
fusion between state and regime, the submerged sense of ethnic grievance across
multiple groups and the Soviet state’s overreach abroad. Moreover, patterns of
nationalist mobilisation broadly reflected such factors as the degree of urbanisation of a
nationality, the size of an ethnic group, its ethnofederal status and the degree to which
it had been assimilated to the dominant Russian culture. But the specific events that
transformed institutions and brought movements to power also contained a heavy dose
of contingency, and their outcomes were hardly predetermined. Repression could
have easily shut down challengers in 1988 and early 1989. At other moments the
backlash effects of repression, the outrage that erupted from intergroup violence and
the anger that materialised out of callous government responses to popular demands
played an important role in transforming the opinion climate of politics and affecting
the prisms through which individuals related to the state and to others. Indeed, those
who organised challenges often sought to provoke responses from states or other
groups that heightened a sense of conflict and identity, so as to drive the engine
of history more quickly, while other movements sought to ride the momentum

24 Bohdan Nahaylo, ‘The Birth of an Independent Ukraine’, Report on the USSR, 3, 50 (13 December
1991), 1–2.

25 See Matthew Wyman, Public Opinion in Postcommunist Russia (New York: St. Martin’s Press, 1997),
166.

Nationalism and the Collapse of Soviet Communism 345

generated from the successful prior actions of others. In short, the agency of ordinary
people needs to be placed squarely in the centre of any accurate understanding of
communism’s collapse.

As E. H. Carr noted, in real life there is no contradiction between the influence of
structure and the role of agency, because structure exercises its effects not by rendering
outcomes inevitable, but rather by making action possible, more probable and more
likely to meet with success.26 But it is also true that, as actions accumulate, they can
also exercise a structure-like effect, as having the capacity to render subsequent action
possible, more probable and more likely to meet with success. Rather than simply
being a manifestation of structurally predetermined conditions, the collapse of Soviet
communism materialised over a five-year period of what I have called ‘thickened
history’ – in which events acquired a sense of momentum, transformed identities
and political institutions and increasingly assumed the characteristics of their own
causal structure. As one Soviet journalist put it in autumn 1989, ‘We are living in
an extremely condensed historical period. Social processes which earlier required
decades now develop in a matter of months.’27 This heightened pace of contention
affected both governing and governed – the former primarily in the state’s growing
incoherence and inability to fashion relevant policies, the latter by introducing
an intensified sense of contingency, possibility and influence from the example of
others.

One of the characteristic features of ‘thickened’ history is that the pace of events
outstrips the movement of institutions and the understanding of leaders. In the
collapse of communism the pace of events was itself a causal factor in the outcome,
as events simply moved far faster than institutions were capable of reacting. This
was most glaringly evident with regard to nationalities issues, in which formulas
embraced by Gorbachev in 1988, 1989 and 1990 soon grew outdated as a result
of shifting events on the ground. The tide of nationalism also produced enormous
confusion and division within Soviet institutions, making it even more difficult to
find institutional solutions to the challenge of holding the Soviet state together. The
pull of alternative movements within the Communist rank-and-file was particularly
strong in many parts of the country. In the course of 1989 nationalist movements came
to dominate republican politics in the Baltic republics, Georgia and Armenia, so that
party organisations largely went ‘underground’, as one communist official put it. In
the 1990 republican elections nationalist movements or those sympathetic with them
came to power in practically every republic with the exception of Azerbaijan and
those in Central Asia, institutionalising the waves of nationalism that had swept across
the country. This was soon followed by what came to be known as the ‘war of the
laws’ – a struggle between the centre and the republics over whose laws actually were
sovereign. Gorbachev insisted that the central government’s laws had precedent over
republics and localities, and declared invalid a whole series of laws that contradicted

26 Edward Hallett Carr, What Is History? (New York: Alfred A Knopf, 1962), 124.
27 Literaturnaia gazeta, 13 September 1989.

346 Contemporary European History

all-union laws. In turn many republics refused to recognise the authority of the centre
over them. The conservative reaction to this disorder pushed the Soviet state towards
its final, tumultuous demise in the failed August 1991 coup.

Nationalist mobilisation not only undermined the authority of state institutions;
it also helped to dissipate the state’s capacity to repress. In the wake of the April 1989
Tbilisi events, the use of the Soviet army as a tool to contain ethnic revolt grew
heavily politicised, and as authority shifted to the republics, actions by the central
government’s institutions of order to quell the nationalist unrest became embroiled
in controversy. The constant deployment of the military and special police units to
nationalist ‘hot spots’ around the country bred a sense of exhaustion among them. The
declining morale of those charged with keeping order was a constant theme during
these years, and over the course of 1990–1 discipline within the armed forces began to
unravel in a serious way. Most of the officers who commanded key units during the
August 1991 coup had been intimately involved in putting down nationalist unrest
in various parts of the country. Given the effect that many of these earlier actions had
on morale within the police and the military, it hardly seems accidental that these
same officers, when called on to use force against a civilian population of their own
nationality on an even larger scale for the sake of preserving the USSR, refused to
carry out their superiors’ orders.

Nationalism within and beyond the collapse

It would be impossible to understand post-communist politics today without
reference to the national dimension of the communist collapse – one of the reasons
why any serious discussion of communism’s demise needs to explicate nationalism’s
role in this process rather than treat it merely as a consequence of the collapse. A
glance at the front page of a randomly chosen Russian newspaper almost two decades
after the collapse included stories about continuing conflicts between Estonia and
Russia over demarcation of their borders, claims by Moscow mayor Yuri Luzhkov that
Ukrainian authorities were discriminating against the Russian-speaking population
of Ukraine, and the opening of regular passenger ship routes between Russia and
Abkhazia despite Georgian objections that this was a gross violation of Georgian
sovereignty.28 The collapse of Soviet communism remains a fundamental reference
point, both positive and negative, for populations throughout the region. It is either
mourned as, in Putin’s words, ‘the greatest geopolitical catastrophe of the century’29

or celebrated as the foundation of a national political community and marked as
a national holiday by fireworks and military parades (as in Ukraine). These identity
narratives are woven into the fabric of new national histories and continue to manifest
themselves politically in issues such as NATO and EU expansion, the geopolitics of

28 Vremia, 2 July 2008, 1.
29 Poslanie Prezidenta Rossiiskoi Federatsii V. V. Putina Federal’nomu Sobraniiu Rossiiskoi Federatsii, 25

April 2005, available at www.kremlin.ru/appears/2005/04/25/1223_type63372type63374type82634_
87049.shtml.

Nationalism and the Collapse of Soviet Communism 347

energy, policies toward Russians and Russophones living in the post-Soviet republics
and desires for and fear of a resuscitation of Russian power in the region. Thus
not only did nationalism occupy a central role in way in which the collapse of
communism unfolded, but the fundamental identity conflicts that gave structure to
the collapse remain with us, manifested now more in the realm of interstate relations,
but nevertheless still central to the ways in which individuals understand themselves
and their relationship to political authority.

  • p1.Rem
  • p2. Rem
  • p.3 Rem
  • p.4 Rem
  • p.5 Rem
  • p.6 Rem
  • p.7 Rem
  • p.7 Rem

  • p.8 Rem
  • p.9 rem

SECESSIONISM FROM THE
BOTTOM UP

Democratization, Nationalism, and Local
Accountability in the Russian Transitio

n

By ELISE GIULIANO*

IN the early 1990s Russia stood at the precipice of state failure. De-mands for autonomy radiated from Russia’s ethnic republics, threat-
ening to split the federation along ethnic lines as had happened to the
Soviet Union before it. Russia’s republics had begun appropriating
power from Moscow during the late Soviet era. With the collapse of
the USSR, several accelerated their quest, asserting control over natural
resources, defying federal laws, and introducing republican presiden-
cies. The decisions some republics made to boycott federal elections
and stop paying federal taxes lent momentum to a process that seemed
likely to end in Russia’s disintegration.

Events in Russia, together with those in Eastern Europe and the
Soviet Union, have reanimated a discussion about the causes of seces-
sionism and the relationship of secessionism to ethnic confl ict and state
collapse. Some scholars argue that secessionism develops in ethnic fed-
erations because their institutional structure generates identities, actors,
and resources that facilitate ethnic mobilization and weaken the central
state. Ethnofederalism,1 in this view, caused Yugoslavia, the USSR, and

* I would like to thank participants at the Comparative Politics Workshop at the University of
Chicago,the Post-Communist Politics and Economics Workshop at Harvard University, and the
Laboratory in Comparative Ethnic Processes for helpful comments on earlier versions of this article.
I acknowledge Dawn Brancati, Stephen Hanson, Harris Mylonas, Josh Tucker, Pieter Van Houten,
Lucan Way, and, in particular, Yoshiko Herrera and the anonymous reviewers for excellent sugges-
tions. I am grateful to Mark Beissinger, Dmitri Gorenburg, and Jeff Kahn for sharing their data. Fi-
nancial support for this article was provided by Columbia University’s Harriman Institute, the Kennan
Institute, the University of Notre Dame’s Kroc Institute, Harvard University’s Davis Center for Rus-
sian and Eurasian Studies, the American Political Science Association, and the International Research
and Exchanges Board.

1 Philip Roeder popularized this term in Roeder, “Soviet Federalism and Ethnic Mobilization,”
World Politics 43 ( January 1991).

World Politics 58 ( January 2006), 276–310

2 Valerie Bunce, Subversive Institutions (New York: Cambridge University Press, 1999); Svante E.
Cornell, “Autonomy as a Source of Confl ict: Caucasian Confl icts in Theoretical Perspective,” World
Politics 54 ( January 2002); Carol Skalnik Leff, “Democratization and Disintegration in Multinational
States: The Breakup of the Communist Federations,” World Politics 51 ( January 1999); Juan Linz and
Alfred Stepan, “Political Identities and Electoral Sequences: Spain, the Soviet Union, and Yugosla-
via,” Daedalus 122 (Spring 1992); Roeder (fn.1); idem, “The Triumph of Nation-States: Lessons from
the Collapse of the Soviet Union, Yugoslavia and Czechoslovakia,” in Michael McFaul and Kathryn
Stoner-Weiss, eds., After the Collapse of Communism: Comparative Lessons of Transition (New York:
Cambridge University Press, 2004); Ronald Suny, The Revenge of the Past: Nationalism, Revolution and
the Collapse of the Soviet Union (Stanford, Calif.: Stanford University Press, 1993).

3 Nancy Bermeo, “The Import of Institutions,” Journal of Democracy 13, no. 2 (2002); Michael
Hechter, Containing Nationalism (New York: Oxford University Press, 2000). Henry Hale makes the
more fi ne-grained argument that ethnofederal collapse is more likely when states contain a core ethnic
region. See Hale, “Divided We Stand: Institutional Sources of Ethnofederal State Survival and Col-
lapse,” World Politics 56 ( January 2004).

4 In practice, however, the autonomy of all ethnic regions was very limited. Ruth Lapidoth, Au-
tonomy: Flexible Solutions to Ethnic Confl icts (Washington, D.C.: United States Institute of Peace Press,
1996).

5 Thus, I exclude republics that were elevated from autonomous oblast to republic in 1991: Ady-
gei, Gorni Altai, Khakassia, and Karachai-Cherkessia. I also exclude Ingushetia, which split off from
Checheno-Ingushetia in 1992. These republics displayed very little separatism and therefore add no
variation to the original sixteen.

Czechoslovakia to rupture along ethnic lines.2 Others scholars disagree,
maintaining that, to the contrary, ethnofederalism reduces confl ict and
“preserves peace.”3 Because some of Russia’s ethnic regions demon-
strated separatism while others remained quiescent, examining Russia
offers an opportunity to better specify whether and how ethnic federa-
tions promote regional secessionism.

This article asks why secessionism emerged in Russia and why cer-
tain republics were more secessionist than others. It compares Russia’s
sixteen autonomous republics (ARs) that were ranked just below the union
republics (URs) in the USSR’s ethnoterritorial administrative hierar-
chy. Soviet leaders established the ARs as homelands for certain ethnic
groups though large populations of ethnic Russians lived in the repub-
lics as well. Russia’s ARs looked considerably more like states than the
lower-ranked autonomous oblasts and autonomous okrugs, insofar as they
had their own legislatures, executives, and judiciaries, as well as fl ags,
constitutions, and some national language education. In addition, com-
pared with lower-ranked territories, ARs were allowed greater, albeit
symbolic, representation in the federal government and limited rights
to set local administrative policy.4 I compare only those regions that
held the status of autonomous republic before 1991, in order to hold
constant these factors of rights, privileges, and institutional develop-
ment.5 I also focus on the ARs because they, like Russia’s oblasts, were
politically more important than lower-level ethnic territories.

S E C E S S I O N I S M F R O M T H E B O T T O M U P 277

Post-Soviet political science has developed a conventional wisdom—
which we may call the “wealth hypothesis”—that explains secessionism
in Russia by focusing on structural economic conditions in the repub-
lics. According to this approach, leaders of resource-rich, economically
developed republics were motivated by their republics’ wealth to make
separatist demands on Moscow. Leaders of comparatively poor repub-
lics, by contrast, lacked such motivation.6 This argument elucidates a
general logic in which structural economic conditions induce actors to
support secession when they expect to profi t from it.7 The logic of the
wealth hypothesis seems persuasive. Yet not all wealthy republics were
secessionist, whereas some poor republics were. The variable of wealth,
therefore, cannot account for separatism among Russia’s republics.8
Moreover, by focusing solely on economic structure, the wealth hy-
pothesis fails to consider how the massive political transformations oc-
curring at the time affected relations between Moscow and republican
leaders and relations among actors within the republics. The approach
developed in this article brings politics back into the study of republi-
can secessionism.

I argue that disintegration of the Soviet Union’s centralized, single-
party system produced a contest for political control inside Russia’s re-
publics—and that that contest determined the strength of separatist
demands the republics made on Moscow. With the massive transfor-

6 The three main comparative studies on secessionism in the Soviet republics are Kisangani Emizet
and Vicki Hesli, “The Disposition to Secede: An Analysis of the Soviet Case,” Comparative Political
Studies 27 ( January 1995); Daniel S. Treisman, “Russia’s ‘Ethnic Revival’: The Separatist Activism of
Regional Leaders in a Postcommunist Order,” World Politics 49 ( January 1997); and Henry Hale, “The
Parade of Sovereignties: Testing Theories of Secession in the Soviet Setting,” British Journal of Political
Science 30 ( January 2000). See also Kathryn Stoner-Weiss, “Federalism and Regionalism” in Stephen
White, Alex Pravda, and Zvi Gitelman, eds., Developments in Russian Politics 4 (Durham, N.C.: Duke
University Press, 1997), 239.

7 This logic was originally delineated by Ernest Gellner, Donald Horowitz, Michael Hechter, Peter
Gourevitch, and Tom Nairn. Gourevitch and Nairn hypothesize that relatively economically advanced
ethnic elites in politically peripheral regions support secessionism to develop their regions’ potential.
Horowitz and Hechter argue, by contrast, that relative economic backwardness inspires ethnic groups
to increase their region’s prospects through secession. See Ernest Gellner, Thought and Change (Chi-
cago: University of Chicago Press, 1964), 147–78; idem, Nations and Nationalism (Oxford: Blackwell
Press, 1983); Donald Horowitz, “Patterns of Ethnic Separatism,” Comparative Studies in Society and
History 23 (1981); idem, Ethnic Groups in Confl ict (Berkeley: University of California Press, 1985);
Michael Hechter, Internal Colonialism: The Celtic Fringe in British National Development, 1536–1966
(Berkeley: University of California Press, 1975); idem, “Group Formation and the Cultural Division
of Labor,” American Journal of Sociology 84, no. 2 (1978); Peter Gourevitch, Paris and the Provinces
(Berkeley: University of California Press, 1980); Tom Nairn, The Break-up of Britain: Crisis and Neo-
Nationalism (London: NLB, 1977).

8 In the same vein, Yoshiko Herrera’s study of sovereignty movements in Russia fi nds that regional
wealth did not drive campaigns for sovereignty among Russia’s nonethnic regions in the early 1990s.
Instead, in certain oblasts, economic interests were constructed in ways that produced demands for
sovereignty. See Herrera, Imagined Economies: The Sources of Russian Regionalism (New York: Cam-
bridge University Press, 2005).

278 W O R L D P O L I T I C S

mations of late perestroika, communist leaders in the republics sud-
denly found themselves accountable to local constituencies rather than
to Moscow. Three key aspects of democratization at the center pro-
duced this shift in accountability. First, the dissolution of the Com-
munist Party (CPSU) ended the nomenklatura system in which regional
leaders were appointed according to party lists maintained by Moscow.
Second, the fi rst semicompetitive elections to republican parliaments
were held in 1990, introducing new representatives with new ideas into
what had been rubber-stamp legislatures. Third, Gorbachev’s glasnost
allowed informal organizations to emerge and employ grassroots tac-
tics such as mass demonstrations. In certain republics, the most popu-
lar, visible, and vocal of these organizations became ethnonationalist
movements, which developed into a critical, yet commonly overlooked
variable in Russia’s transition. Together then, these three aspects of lib-
eralization transformed Russia’s republics very quickly from hollow ad-
ministrative units into new competitive arenas or, in the words of Carol
Leff, into “separate and distinct political marketplaces.”9

Once we recognize that change in the center restructured the incen-
tives of republican leaders by creating a contest for local control, it is
possible to theorize how relations among subfederal actors—nomen-
klatura leaders, opposition nationalist movements, and mass publics—
infl uenced secessionism vis-à-vis Moscow. As glasnost progressed in
the late 1980s, popular opinion in several of Russia’s republics began to
support nationalist programs of increased rights for titular nationalities
and greater republican autonomy.10 In those republics where national-
ist movements were gaining popularity, the very visible fact of growing
crowds at rallies and the rising status of opposition leaders on the street
and in local parliaments represented a palpable threat to incumbents.
Communist leaders were pressured into addressing nationalist pro-
grams and petitioning Moscow for autonomy. In other republics, na-
tionalists replaced nomenklatura leaders and led campaigns for sover-
eignty themselves. These scenarios occurred in republics that mounted
the strongest challenges to federal authority: Tatarstan, Tuva, Chech-
nya, Yakutia, and Bashkortostan.11 Conversely, in republics where na-
tionalist movements failed to attract support, incumbents could ignore
nationalist appeals with impunity and remain safely in offi ce.

9 Leff (fn. 2), 210.
10 The term titular denotes the ethnic group for which the republic is named, for example, Tatars

in Tatarstan.
11 I use the post-Soviet names of Russia’s republics throughout this article for consistency, except

when discussing Checheno-Ingushetia before Ingushetia separated from the republic in 1992.

S E C E S S I O N I S M F R O M T H E B O T T O M U P 279

I argue that variation in popular support for nationalism can explain
the level of republican secessionism toward Moscow. This variable can
account for cases that the wealth hypothesis is at a loss to explain. While
the secessionist republics of Tatarstan, Yakutia, and Bashkortostan were
relatively resource rich, the fact of secessionism in Tuva—a tiny, poor
republic whose economy centered on livestock herding—fl atly contra-
dicts the wealth hypothesis. Likewise, secessionism in Chechnya can-
not be explained by the wealth hypothesis. While Chechnya contained
some crude oil and an oil pipeline connecting Russia with refi neries
in Baku, its reserves made up a minuscule 1 percent of Russia’s to-
tal output in 1992, and its pipeline was threatened with redundancy
by new pipeline projects already under way that bypassed the repub-
lic.12 Chechnya and Tuva rank among Russia’s poorest regions, whether
measured in terms of raw materials, population size, standard of liv-
ing, or industrial production—yet they were among the most seces-
sionist.13 Moreover, the presence of rich natural resources did not spur
separatism in Komi—a highly industrialized republic that contained
enormous coal deposits and signifi cant oil and gas fi elds. The fact that
the wealth hypothesis can explain neither separatism in the poor re-
publics nor its absence in the rich ones14 suggests that we need another
explanation for variation in republican secessionism. Popular support
for nationalism, I will show, was present in secessionist republics but
missing in quiescent ones.

If popular nationalism was critical in driving republican secession-
ism, where did it come from? Did incumbent leaders foment mass na-
tionalism in order to strengthen their republic’s negotiating position
with Moscow?15 If so, this would mean that incumbents were able to
act autonomously at this time of unprecedented political ferment. Yet
Gorbachev’s policies drained power from the Communist Party, intro-

12 Anatoly Khazanov, After the USSR: Ethnicity, Nationalism and Politics in the Commonwealth of
Independent States (Madison: University of Wisconsin Press, 1995), 219.

13 Oksana Genrikhovna Dmitrieva, Regional’naia ekonomicheskaia diagnostika (Regional economic
diagnostics) (St. Petersburg: Izdatel’stvo Sankt-Peterburgskogo Universiteta Ekonomiki i Finansov,
1992), 128–32.

14 Similarly, Russia’s wealthiest ethnic regions—the autonomous okrugs of Khanty-Mansisk and
Yamal-Nenets—also failed to make separatist demands. Though these regions, as okrugs, had fewer
institutions, rights, and privileges than republics, they were far richer, producing 80 percent of Russia’s
oil and gas. Roy Bahl and Christine I. Wallich, “Intergovernmental Fiscal Relations in the Russian
Federation,” in Richard M. Bird, Robert D. Ebel, and Christine I. Wallich, eds., Decentralization of
the Socialist State (Washington, D.C.: World Bank, 1995), 326. After federal relations had stabilized
in the late 1990s, okrug leaders demanded administrative independence from Tyumen oblast to retain
more control over natural resources. See “Khanty-Mansi Avtonomnyi Okrug,” in Regiony Rossii: statis-
ticheskii sbornik (Russia’s regions: Statistical handbook) (Moscow: Goskomstat, 1999).

15 This argument can be found in Roeder (fn. 2); Treisman (fn. 6); and Jeff Kahn, Federalism, De-
mocratization, and the Rule of Law in Russia (New York: Oxford University Press, 2002), 126–32.

280 W O R L D P O L I T I C S

duced competitive elections, and allowed opposition groups to form, all
of which eroded the autonomy of incumbents within their republics.
Moreover, the view that republican leaders effortlessly manipulated
masses into a nationalist fervor treats elites as strategic actors but mass
populations as passive instruments who were either (1) “genuine” na-
tionalists, patiently waiting for their chance at statehood or (2) Soviet
subjects, dutifully obeying their leaders’ commands.16 This approach
essentializes ethnic populations and overestimates the power of ethnic-
ity as a basis of political action. Instead, republican residents, like citizens
in Russia’s nonethnic regions, behaved rationally, backing one or another
politician and occasionally shifting or withdrawing their support.

What, then, were the origins of popular support for nationalism?
The explanation offered in this article focuses on how increasing com-
petition for jobs in the Soviet Union’s failing economy allowed par-
ticular issues articulated by nationalist leaders to resonate with ethnic
populations. Nationalists claimed that their ethnic groups were denied
full participation in republican economies due to blocked access to de-
sirable jobs and resources. They deplored titulars’ socioeconomic sub-
ordination to Russians, blamed a discriminatory central state, and ar-
gued that achieving republican sovereignty would eliminate oppression.
However, the nationalists’ claims were at odds with the fact that titulars
had made astonishing professional gains under Soviet rule due to of-
fi cial policies of affi rmative action. But the way in which the nation-
alists framed issues of ethnic economic inequality resonated because
it provided people with an interpretation of their current experience
of rising competition for jobs in an exceptionally insecure environ-
ment. Nationalist leaders politicized ethnicity by convincing people to
connect personal material interests to one of their social identities—
ethnicity. They persuaded people that their personal life chances de-
pended on the political fate of their ethnic community.

This article is structured as follows. Section I defi nes the dependent
variable by introducing a new coding of secessionism in Russia’s sixteen
republics. Sections II and III explain (1) the origins of popular nation-
alism within the republics and (2) the infl uence of popular nationalism
on republican secessionism from 1989 to 1994. In Section II, evidence
presented for the origins of mass nationalism is based on analysis of

16 Dmitri Gorenburg’s work is an exception. He shows how local institutions produced variation
in mass nationalist mobilization across Russia’s republics, in Gorenburg, Minority Ethnic Mobilization
in the Russian Federation (New York: Cambridge University Press, 2003). Hale recognizes that masses
may have different preferences than leaders, but he posits that macroeconomic factors infl uenced all
actors in the republics uniformly. See Hale (fn. 6).

S E C E S S I O N I S M F R O M T H E B O T T O M U P 281

data on the ethnic composition of republican workforces, as well as on a
discourse analysis of nationalist platforms and local newspaper articles
published in the republics. Section III demonstrates that mass nation-
alism infl uenced secessionism, using data showing correlations between
secessionism, on the one hand, and ethnic demonstrations and ethnic
violent events in the republics, on the other. In addition, case studies
of three republics demonstrate that mass nationalism in Tuva and Ya-
kutia infl uenced republican demands on Moscow at key moments in
their sovereignty campaigns, while low mass nationalism in Mari-El
resulted in low secessionism there. I maximize variation on the inde-
pendent variable of mass nationalism by comparing two cases that are
representative of Russia’s secessionist republics, Tuva and Yakutia, with
Mari-El, a case that typifi es Russia’s low-secession republics. In addi-
tion, the comparison of Tuva and Yakutia casts doubt on the wealth
hypothesis insofar as these republics differ in terms of wealth (Tuva is
poor; Yakutia is wealthy) but share a common outcome: secessionism.
Section IV offers conclusions.

I. SECESSIONISM AMONG RUSSIA’S REPUBLICS

To determine levels of secessionism among Russia’s republics, I use a
fourteen-point coding index based on actions initiated by the repub-
lics, as well as on their responses to Moscow’s policies during the years
1989–94. My coding captures the main ways in which the republics
challenged Moscow and improves upon previous codings that have
relied on a single indicator, omitted key developments in center-
periphery relations, and miscoded the critical case of Tuva.17 The indica-
tors (discussed below; and listed in Table 1) show that the most secessionist
republics were Tatarstan, Tuva, Chechnya, Bashkortostan, and Yakutia.18

In the earliest separatist act, all of the autonomous republics issued
declarations of sovereignty following the Russian Soviet Federated So-
cialist Republic’s (RSFSR) momentous decision to do so in June 1990
(indicator 1). For the next year and a half Gorbachev tried to preserve
the Soviet Union and undermine the growing power of Boris Yeltsin by
offering more rights to the autonomous republics, even suggesting that

17 See Emizet and Hesli’s (fn. 6) use of the timing of Supreme Soviets’ sovereignty declarations as a
single indicator of secessionism (p. 500); and Treisman’s (fn. 6) miscoding of Tuva (pp. 224–25).

18 Leading Russian specialists also identify these republics as the most separatist. See Valery Tish-
kov, Ethnicity, Nationalism and Confl ict in and after the Soviet Union (London: Sage Publications,
1997), 58; Leokadia Drobizheva, Natsional’noe samosoznanie i natsionalizm v Rossiiskoi Federatsii nach-
ala 1990-x godov (National consciousness and nationalism in the Russian Federation in the early
1990s) (Moscow: Institute of Ethnography and Anthropology, Russian Academy of Sciences, 1994).

282 W O R L D P O L I T I C S

TABLE 1
INDEX OF SECESSIONISM IN RUSSIA’S REPUBLICS

(1989–94)

1. Declared sovereignty 1 1 1 1 1 1 1 1 1 1 1 1 1 1 1 1
2. Boycotted RSFSR 1 1 1 1
presidency referendum,
1991
3. Established presidency 1 1 1 1 1 1 1 1 1 1 1 1
4. Passed language law 1 1 1 1 1
5. Titular language sole 1
language
6. Set own tax policy/ 1 1 1 1 1 1
stopped paying taxes
7. Refused to sign Yeltsin’s 1 1
Federation Treaty, 1992
8. Held sovereignty 1 1
referendum
9. Adopted constitution 1 1 1 1 1 1 1 1 1 1 1 1
10. Before new RF 1 1 1
constitution?
11. With right to secede? 1 1
12. Republic law supreme? 1 1 1 1 1 1 1 1
13. Boycotted/ invalid 1 1
referendum on Yeltsin,
April 1993
14. Boycotted referendum 1 1
on RF constitution,
Dec. 1993
Index of Secessionism 12 10 9 8 6 4 4 4 4 3 3 3 2 2 2 1

T
at

ar
st

an

T
uv

a

C
he

ch
ny

a

Y
ak

ut

ia

B
as

hk
or

to
st

an

K
a

l

m
yk

ia

K
ar

el
ia

B
ur

ya
t

i

a

N
. O

ss
et

ia

K
om

i

K
ab

ar
di

no
-B

al
ka

ri
a

C
hu

va
sh

ia

U
dm

ur
ti

a

D
ag

es
ta

n

M
ar

i-
E

l

M
or

do
va

their status in a “renewed federation” would match that of the Union
republics.19 Yeltsin also promised the ARs greater recognition within
a sovereign Russia, telling them, “Take as much sovereignty as you
can handle.”20 Despite Yeltsin’s efforts, fewer voters in the republics
than in the rest of the RSFSR voted to create a Russian (read: Yeltsin)
presidency in a March 1991 referendum. Tatarstan, Tuva, Checheno-
Ingushetia, and North Ossetia even refused to hold the referendum
(indicator 2).21

After the Soviet collapse, the behavior of the republics diverged.
Some scaled back their separatist activity. Others took advantage of
central state weakness by establishing presidencies (indicator 3), draft-
ing constitutions, and passing language laws (indicator 4), some of
which named the titular language as the sole offi cial state language
(indicator 5). When Yeltsin’s program of shock therapy led to rampant
infl ation and reduced central fi nancing of regional budgets, a few re-
publics began to withhold tax revenue and/or set their own tax policy.22
Tatarstan, Tuva, and Chechnya, for example, “virtually ceased paying
their taxes in 1993,” while Yakutia and Bashkortostan regularly delayed
remitting taxes to Moscow (indicator 6).23

A constitutional crisis cum power struggle between the executive
and legislative branches of Russia’s federal government developed,
prompting Yeltsin to try to secure the republics’ support by drafting a
Federation Treaty that was to become part of the new Russian constitu-
tion. The Federation Treaty acknowledged republican sovereignty and
granted special rights to ethnic regions. Though most republics voted
the treaty into law in March 1992, Tatarstan and Chechnya refused to

19 Thus, a majority of the electorate in the ARs (82.6 percent) voted to preserve the USSR in the
March 1991 referendum. See Edward Walker, Dissolution: Sovereignty and the Breakup of the Soviet
Union (New York: Rowman and Littlefi eld, 2003), 96, 102, 117; Vladimir Shlapentokh, Roman Lev-
ita, and Mikhail Loiberg, From Submission to Rebellion: The Provinces versus the Center in Russia (Boul-
der, Colo.: Westview Press, 1997), 91; and Yurii Baturin, “Shakhmatnaia diplomatia v Novo-Ogarevo,”
Democratizatsia 2 (Spring 1994).

20 E. Chernobrovkina, “Reshat’ vam samim,” Vechernaia kazan (August 10, 1990), 1.
21 Ann Sheehy, “The All-Union and RSFSR Referendums of March 17,” RFE/RL Research Report

3 (March 29, 1991), 22. Results of the presidential election in June 1991 indicate the same pattern:
Yeltsin won 50.6 percent of the votes in the autonomous regions compared with 58.4 percent in the
RSFSR as a whole. Walker (fn. 19), 117, 123–24.

22Elizabeth Teague, “Center-Periphery Relations in the Russian Federation,” in Roman Szpor-
luk, ed., National Identity and Ethnicity in Russia and the New States of Eurasia (Armonk, N.Y.: M. E.
Sharpe, 1994), 42; and Darrel Slider, “Federalism, Discord, and Accommodation: Intergovernmental
Relations in Post-Soviet Russia,” in Theodore Friedgut and Jeffrey Hahn, eds., Local Power and Post-
Soviet Politics (Armonk, N.Y.: M. E. Sharpe, 1994), 249.

23 Ingushetia and North Ossetia declared bankruptcy and also reduced tax payments to Mos-
cow; Shlapentokh, Levita, and Loiberg (fn. 19), 169. See also Jeremy Azrael and Emil Payin, Confl ict
and Consensus in Ethno-Political and Center-Periphery Relations in Russia (Santa Monica, Calif.: Rand
Conference Proceedings, 1998), 29.

284 W O R L D P O L I T I C S

sign, and Bashkortostan signed only after Moscow added an appendix
recognizing its independence. Yakutia also took an oppositional stance
throughout the negotiations and signed after obtaining an agreement
allowing it to retain part of the revenue from diamonds and gold on its
territory (indicator 7).24

At about the same time, certain republics initiated highly destabi-
lizing acts. In Tatarstan 61 percent of the population voted yes in a
referendum on state sovereignty that many interpreted as a vote for
independence.25 Bashkortostan held a similar, but less radical referen-
dum a year later (indicator 8). In another series of challenges, most re-
publics adopted constitutions (indicator 9). Tatarstan, Tuva, Chechnya,
and Yakutia did so prior to the December 1993 referendum on Russia’s
federal constitution organized by Yeltsin (indicator 10).26 The boldest
republican constitutions contested Moscow’s authority by stipulating
that republican law took supremacy over federal law and specifi ed the
right to secede in the case of Tuva (indicators 11 and 12).27

In a critical event in April 1993, Yeltsin held a national referendum
on his leadership and reformist policies in order to delegitimize the
antireform federal legislature. A majority of Russia’s population voted
for Yeltsin, but support varied in the republics, with six of the sixteen
voting against him.28 Two republics, Tatarstan and Chechnya, blatantly
defi ed federal rule by refusing to hold the referendum on their terri-
tory (indicator 13).29 The republics maintained the initiative in rela-
tions with the center until September 1993, when Russia’s constitu-
tional crisis came to a head and Yeltsin rashly passed a decree dissolving

24 Teague (fn. 22); Vera Tolz, “Thorny Road toward Federalism in Russia,” RFE/RL Research Report
2 (December 3, 1993); and Kahn (fn. 15), 126–32.

25This perception was reinforced ten days later when Tatarstan refused to sign the Federation Treaty.
Ann Sheehy, “Tatarstan Asserts Its Sovereignty,” RFE/RL Research Report 1, no. 203 (April 3, 1992), 1.

26 In fact, the actions of these four republics convinced Moscow of the need to replace the Federa-
tion Treaty with a new federal constitution; Tishkov (fn. 18), 62.

27 Most constitutions asserted control over natural resources and gave both Russian and the titular
language offi cial status. See Teague (fn. 22), 43; Kahn (fn. 15), 82–84; and Gorenburg (fn. 16). For
the full text of the constitutions, see Iu. A. Dmitriev and E. L. Malakhova, Konstitutsii respublik v sos-
tave Rossiiskoi Federatsii (Republican constitutions of the Russian Federation) (Moscow: Izdatel’skaia
Firma Manuscript, 1995).

28 Bashkortostan, Dagestan, Kabardino-Balkaria, Mari-El, Chuvashia, Mordova, and Ingushetia
voted against Yeltsin. In Tuva a majority endorsed him in questions one and two, but voted for early
presidential elections—a rejection of Yeltsin. Wendy Slater, “No Victors in the Russian Referendum,”
RFE/RL Research Report 2 (May 21, 1993). However, an anti-Yeltsin vote was not necessarily a chal-
lenge to federal authority, since some republics voted against him to signal support for the Supreme
Soviet’s conservative policies of preserving federal subsidies to the regions. Ralph Clem and Peter
Craumer, “The Geography of the April 25 (1993) Russian Referendum,” Post-Soviet Geography 34
(October 1993).

29 Chechnya boycotted the referendum, while Tatarstan’s administration discouraged voting, re-
sulting in a 22.6 percent voter turnout, which invalidated the republic’s results.

S E C E S S I O N I S M F R O M T H E B O T T O M U P 285

parliament. Several republics condemned or ignored his decree during
the two-week standoff in which deputies refused to leave the parlia-
ment building in Moscow. A majority of republics, however, softened
their stance following Yeltsin’s drastic decision to shell parliament and
arrest the opposition. Most republics complied with his ensuing order
to hold new local parliamentary elections, but Tatarstan and Chechnya
obdurately boycotted a national referendum on Russia’s constitution
(indicator 14).3

0

Some analysts believe Yeltsin’s dissolution of parliament averted
Russia’s collapse. Whether or not that belief is accurate, it did permit
the central government to consolidate power, beginning with the pas-
sage of the Russian Constitution. Another important development was
Moscow’s decision to sign bilateral treaties with several republics. Yet
by the time this occurred in the mid-1990s, mass nationalism was fad-
ing in even the most assertive republics. Nationalist movements had
been losing support, in some cases as early as 1993. This suggests that
declining subfederal political competition led to lower levels of seces-
sionism vis-à-vis Moscow. By the mid-1990s republican separation
from Russia had become unlikely. Yeltsin’s centralization of power al-
tered Russia’s entire institutional environment, shifting power from re-
publican parliaments to executives and eliminating the massive central
state weakness that had made possible republican challenges to federal
sovereignty in the early 1990s.

Coding secessionist activity toward Moscow raises the question of
whether the republics desired independence or political autonomy.
Some observers differentiate between the two, arguing that Russia’s
republics sought mere autonomy all along. They accurately note that
only Chechnya issued an actual declaration of independence. Yet to
conclude that the other republics wanted autonomy rather than inde-
pendence would be to deduce actors’ intentions at that time by observ-
ing the fi nal outcome of nonsecession. This approach, I argue, misun-
derstands the dynamic and political nature of separatist campaigns. In-
tentions of regional actors cannot be gleaned directly from their state-
ments and tactics because those intentions shift and evolve along with
political opportunities. As Donald Horowitz states, actors’ “[d]emands
can switch from autonomy to independence and back again, depending

30 Seven republics voted for the constitution, fi ve voted against it, and voter turnout was under 50
percent in two republics. Gail W. Lapidus and Edward W. Walker, “Nationalism, Regionalism, and
Federalism: Center-Periphery Relations in Post-Communist Russia,” in Gail Lapidus, ed., The New
Russia: Troubled Transformation (Boulder, Colo.: Westview Press, 1995), 100–101. On local elections,
see Elizabeth Teague, “North-South Divide: Yeltsin and Russia’s Provincial Leaders,” RFE/RL Research
Report 2 (November 26, 1993).

286 W O R L D P O L I T I C S

on the state of negotiations between central regimes and separatists.”
The terms secession and separatism should refer not only to indepen-
dence movements, he argues, but also to those “seeking any territorially
defi ned political change intended to accord an ethnic group autono-
mous control over the region in which it resides.”31 In Russia’s republics
moderate appeals for autonomy either led to radical demands or dis-
sipated over time. Therefore, in analyzing secession, the intentions of
regional actors cannot be inferred by assuming that outcomes of non-
secession directly represent their preferences at an earlier point in time.32
Conceptualizing secessionism as a dynamic process permits theorizing
about why demands intensify or slacken. Republican separatism should
be viewed as the evolving product of interaction among various actors
rather than as a fi xed preference of a unitary actor. In Russia secession-
ist demands on Moscow developed from the relationship between the
masses, nationalist movements, and incumbent leaders. Before examin-
ing this relationship in detail, the next section addresses the origins of
mass nationalism in the republics.

II. THE ORIGINS OF POPULAR NATIONALISM

Popular support for nationalism developed in the republics of Tatarstan,
Tuva, Chechnya, Bashkortostan, and Yakutia because, in a declining
economy with increasing competition for jobs, certain issues articulated
by nationalist leaders resonated with ethnic populations fearful about
their life chances. Instead of blaming job competition, however, na-
tionalists decried titulars’ blocked access to jobs and resources. Titulars’
supposed socioeconomic subordination to Russians kept them from
fully participating in the industrialized economy of “their” own repub-
lic. The nationalists blamed this serious injustice on Moscow’s policies
rather than on local Russians. Winning state sovereignty, they argued,
would rectify the injustice.

Paradoxically, the nationalists’ issue frame of ethnic economic in-
equality contradicted the actual situation of rising titular professional-
ization and achievement. At the beginning of the Soviet era, there was
an ethnic division of labor in which titular ethnic groups in Russia’s
republics lived and worked in the countryside while Russians, for the
most part, worked in more highly skilled jobs in the cities. Yet the Soviet

31 Donald Horowitz, “Patterns of Ethnic Separatism,” Comparative Studies in Society and History
23 (April 1981), 168–69.

32 This assumption commits the fallacy of retrospective determinism. I use the terms secession,
separatism, and autonomy campaigns interchangeably throughout this study.

S E C E S S I O N I S M F R O M T H E B O T T O M U P 287

state resolved to advance titular nationalities through specifi c policies
of industrialization, urbanization, public education, Russifi cation, and
indigenization. The policy of indigenization, or korenizatsiia, granted
preferences to titulars in higher education, economic management, and
government administration inside their own republics and sought to
“proletarianize” rural dwellers by moving them into industrializing So-
viet cities.33 Following World War II and accelerating in the 1960s,
titulars took advantage of these quotas, moving from farm to factory
and assuming jobs alongside Russians. By the late 1970s titulars were
working not only in government administration and the Communist
Party but in economic management as well. Thus, Soviet state policies
produced urban, educated ethnic minorities, transforming the lives of
ordinary people and their families in the process.

Meanwhile, ethnic Russians living in the republics were also moving
to the cities as part of the broader trends of urbanization and indus-
trialization occurring in the Soviet Union.34 Thus, by the late 1980s
Russians worked in white-collar jobs in more or less equal proportion
to their share of the total workforce in most republics. The same was
true of titular ethnic groups. Although some of the poor republics still
displayed an ethnic division of labor as of 1989, titulars had been mak-
ing signifi cant strides even in these places. In Tuva, for example, equal
numbers of Tuvans and Russians graduated with higher educational
degrees in 1989, and in Chechnya, Chechens’ rate of higher education
exceeded that of Russians between 1979 and 1989.35

This tremendous social mobility is shown in Figure 1, which pres-
ents a ratio of titular to Russian representation in the white-collar
workforce compared with each group’s percentage of the total work-
force in each republic.36 A ratio of 1 indicates that both titulars and
Russians were equally represented in white-collar jobs in proportion to
their percentage of the total workforce. A ratio greater than 1 means
that titulars held white-collar jobs in numbers greater than their share

33 Yuri Slezkine, “The USSR as Communal Apartment, or How a Socialist State Promoted Ethnic
Particularism,” Slavic Review 53 (Summer 1994), 433. See also Roeder (fn. 1); and Suny (fn. 2).

34 Moshe Lewin describes how these processes transformed the entire Soviet population in Lewin,
The Gorbachev Phenomenon (Berkeley: University of California Press, 1988).

35 The same was true of Yakuts in Yakutia. G. S. Denisova, Etnicheskii faktor v politicheskoi zhizni
Rossii 90-x godov (The ethnic factor in Russia’s political life during the 1990s) (Rostov-on-the-Don:
Rostov State Pedagogical University, 1996), 86–88. See also Leokadia Drobizheva, “Processes of Dis-
integration,” in Vladimir Shlapentokh, Munir Sendich, and Emil Payin, eds., The New Russian Dias-
pora: Russian Minorities in the Former Soviet Republics (Armonk, N.Y.: M. E. Sharpe, 1994), 47.

36 These ratios were compiled by the author based on unpublished raw data collected by Gos-
komstat, the State Statistical Committee of the USSR, as part of the 1989 All-Union census. See
Professional’no-otraslevoi sostav intelligentsia naseleniia titul’noi i russkoi natsional’nostei absolytnie znach-
eniia (Titular and Russian ethnic group composition of white-collar economic sectors). I thank Dmitri
Gorenburg for providing the data.

288 W O R L D P O L I T I C S

of the republican workforce compared with Russians. A ratio less than
1 indicates that as a proportion of each group’s overall representation in
the workforce, titulars were underrepresented compared with Russians.
The average for all republics is 0.814 (see dotted line) which shows that
in a majority of republics, Russians were only slightly more proportion-
ately represented in white-collar jobs than titulars.37 Titulars, as might
be expected, occupied a majority of white-collar jobs in republican rural
areas. But they were also very strongly represented in the coveted up-
per echelons of urban economies and worked as enterprise directors in
numbers equal to or greater than their share of the total population in
almost all republics.38 (See Figure 2.)

Macroeconomic decline that had begun in the Brezhnev era acceler-
ated under Gorbachev. Widespread shortages of consumer goods and
food, followed by runaway infl ation after Yeltsin deregulated prices,
profoundly destabilized the daily lives of Soviet citizens. This means
that demand for white-collar jobs was rising at the same time that the

37 Only in Chechnya and Mari-El do Russians dominate white-collar sectors, although Chechens
constitute 40 percent of Checheno-Ingushetia’s white-collar workforce and 54 percent of its total
workforce. Chechens form 55 percent of the republic’s total population.

38 Mari El and Tuva are exceptions, although Tuvans still constituted 40 percent of directors in
Tuva. See Narodnoe khoziaistvo RSFSR (The economy of the RSFSR) (Moscow: Goskomstat, 1989).

S E C E S S I O N I S M F R O M T H E B O T T O M U P 289

FIGURE 1
RATIO OF TITULAR/RUSSIAN REPRESENTATION IN WHITE-COLLAR WORKFORCE

))

Burya

tia

Kalmkia

North Ossetia

Komi

Kabardino-Balkaria (K/R)

Tatarstan

Yakutia

Bashkortostan (Bash/Rus)

Mordovia

Karelia

Udmurtia

Dagestan

Chuvashia

Tuva

Mari-El

Chechnya

Karbardino-Balkaria (B/R)

0 0.2 0.4 0.6 0.8 1 1.2 1.4

Bashkortostan (Bash/Tatar)

FIGURE 2
TITULAR PERCENTAGE OF ENTERPRISE DIRECTORS COMPARED WITH TITULAR

PERCENTAGE OF POPULATION
(1989)

Soviet economy had stopped growing. People who expected their lives
and those of their children to progress along familiar, state-established
trajectories—secondary school followed by higher education or tech-
nical training, followed by secure jobs and advancement up the career
ladder—became uncertain that the state would continue to provide
them with educations and jobs. Growing labor competition in this cli-
mate caused people to experience material dissatisfaction and fear of
job loss. But if these trends affected all citizens, why would only titu-
lars respond by supporting nationalism? Soviet enterprises did not for-
mally organize job applications or work collectives along ethnic lines.39
And people had class, regional, professional, generational, and national
identities, in addition to ethnic ones. Why would they think of them-
selves as competing along ethnic lines?

In certain republics people began to think of themselves as part of
an ethnonational community because nationalist leaders changed the
meaning of ethnic identity at this time. The way in which national-
ists framed issues about unequal access to resources and jobs helped
to create a sense of groupness connected to specifi c political interests.

39 According to ethnic competition theory, rising job competition may spur mobilization as groups
begin competing for resources in a common economic niche, especially following sudden macroeconomic
change. Frederik Barth, ed., Ethnic Groups and Boundaries (Boston: Little, Brown, 1969); Susan Olzak,
The Dynamics of Ethnic Competition and Confl ict (Stanford, Calif.: Stanford University Press, 1992).

290 W O R L D P O L I T I C S

Ta
rta

rst
an

Tu
va

Ya
ku

tia

Ch
ec

he
no

-I
ng

us
he

tia

Ba
sh

ko
rto

sta
n

Ka
lm

yk
ia

Ka
rel

ia

Bu
ry

ati
a

N
. O

sse
tia

Ko
m

i

Ka
ba

rd
in

o-
Ba

lka
ria

Ud
m

ur
tia

Ch
uv

as
hi

a
D
ag

es
tan

M
or

do
va

M
ar

i-E
l

100

80

60
40
20
0

Titular % of Directors Titular % of Population

What did these issue frames look like? Nationalists claimed that titular
ethnic groups worked in agrarian jobs in the countryside while Rus-
sians occupied prestigious positions in the city. The best jobs requiring
the best educations were “given” to Russians, condemning titulars to a
subordinate social status. It is critical to note that this framing not only
ignored Soviet state policies that promoted titular mobility, but it also
painted titulars as victims of the Soviet—and ethnically Russian—state.
By claiming victim status and blaming the current state, nationalists
could argue that titulars were “subjects” of Russians within their “own”
homelands—a situation violating the basic nationalist principle that
“the political and national unit be congruent.”40 Among the many kinds
of issues articulated by nationalists, those concerning impediments to
ethnic economic equality resonated most strongly because they pro-
vided titulars with a plausible interpretation of their experience of ris-
ing job competition. People came to believe that their life chances were
constrained because of their non-Russian ethnic identity in a Russian
state and began to support the nationalist solution of establishing eth-
nonational state sovereignty.

A comparative discourse analysis of issues articulated by national-
ist leaders in (1) their founding platforms and (2) newspaper articles
published in the republics during the period 1989–94 indicates that the
issue frame of ethnic economic inequality was present in the national-
ist republics of Tatarstan, Tuva, Chechnya, Bashkortostan, and Yakutia
but absent from republics in which titulars failed to back nationalist
movements, such as Mordova and Mari-El.41 My analysis examines
three dimensions of issue framing: how nationalists defi ned a social
condition as “unjust, intolerable, and deserving of corrective action”;
how they attributed blame for the unjust situation to an actor or set of
social processes; and how they identifi ed solutions to that injustice.42
For example, a Yakut nationalist describes inequality among Yakuts and
Russians in the mining industry as unjust, blames the Soviet state for
privileging Russians, and identifi es sovereignty as the solution:

40 Gellner (fn. 7, 1983), 1.
41 Working in two Moscow libraries, I culled through fi ve years of at least two newspapers pub-

lished in each of Russia’s sixteen republics. I collected between forty and one hundred articles from
each republic that address some aspect of ethnic politics. The articles include standard news reports,
offi cial statements by political leaders, editorials, letters to the editors, and statements by nationalist
organizations.

42 The social movements literature identifi es these dimensions as critical. See David A. Snow and
Robert D. Benford, “Master Frames and Cycles of Protest,” in Aldon Morris and Carol McClurg, eds.,
Frontiers in Social Movement Theory (New Haven: Yale University Press, 1992), 137; and David Snow
and Robert Benford, “Framing Processes and Social Movements: An Overview and Assessment,” An-
nual Review of Sociology 26 (2000).

S E C E S S I O N I S M F R O M T H E B O T T O M U P 291

The USSR . . . ignored the equality not only of its citizens, but of entire nations
. . . not only high salaries but special privileges were established for migrants
who came to work in the mining industry, including northern salary supple-
ments, apartments, the best supplies. . . . Only in 1961 was there a partial equal-
ization when salary supplements were given to indigenous residents of Yakutia.
All other main privileges remained. . . . Only sovereignty . . . can regulate and
stabilize relations among people and prevent mass impoverishment of the re-
public’s indigenous residents.43

Similarly, the nationalist Tatar Public Center asserts in its founding
program that the Tatar nation is endangered due to its subordinate sta-
tus vis-à-vis Russians. Since the beginning of the Soviet era and lasting
to the present day, it argues, Tatars outnumbered Russians in rural areas
and exhibited a slower rate of urbanization and “lower qualifi cations.”
The group then advocates specifi c solutions such as regulating the “eth-
nic composition” of higher educational institutions, helping rural Tatars
secure jobs in the republic’s cities and providing Tatars with set-aside
positions in the economy.44

Nationalist leaders, therefore, attracted popular support not by sim-
ply picking up on preexisting attitudes among ethnic populations but
by actively creating grievances that interpreted economic information
in ethnic terms. Nationalists’ attention to these kinds of issues reveals
just how much they were products of the very Soviet state whose au-
thority they were trying to undermine. Their focus on pragmatic issues
of professional mobility, equal ethnic group representation, and indi-
vidual advancement reproduced typical late-Soviet-era ideals. Nation-
alists in the republics saw no contradiction in asserting the right to a
nation-state in order to realize the goods of modernity as defi ned by the
Soviet state. Like all nationalists, they drew on a range of extant beliefs
and practices to construct an idea of national community deserving
control of the state.45

It is critical to note, however, that growing popular support for na-
tionalism does not mean that entire ethnic groups were suddenly won
over to the nationalist cause. On the contrary, support for nationalism
varied greatly—many people were apathetic or hostile. Even within the
most nationalist republics support varied across space, just as in many

43 V. Kopylov, “Kogo pugaet suverenitet?” Yakutia 14 (October 1992), 2.
44 Damir Iskhakov, Sovremennie natsional’nie protsessi v respublike Tatarstan (Current ethnic pro-

cesses in the Republic of Tatarstan) (Kazan: Russian Academy of Sciences, 1992.)
45 This underscores John Breuilly’s point that nationalism is not “the expression of pre-existing

national values and practices in political form.” See Breuilly, Nationalism and the State (Chicago: Uni-
versity of Chicago Press, 1993), 69.

292 W O R L D P O L I T I C S

Union republics.46 Yet despite the ultimately transitory nature of na-
tionalist mobilization in Russia, the fact remains that signifi cant dif-
ferences in levels of popular nationalism existed among the republics.
Though all republics faced similarly fl uid and ambiguous conditions as
a result of central state implosion, only some used the opportunity to
make serious secessionist claims on Moscow. Before showing how mass
nationalism infl uenced separatism, I describe below how democratiza-
tion at the center destroyed vertical accountability and forced republi-
can leaders to respond to local opinion.

III. DEMOCRATIZATION, SHIFTING ACCOUNTABILITY,
AND NATIONALISM

During the Soviet era political accountability ran vertically from cen-
ter to region, with the Communist Party in Moscow appointing re-
publican leaders according to the nomenklatura system. Regional party
leaders were selected and dismissed on the basis of central party de-
cisions.47 These nomenklatura leaders in turn controlled the appoint-
ment of party and managerial cadres within their republics.48 But Gor-
bachev embarked on a strategy of shifting power away from the CPSU
and toward the state, that is, toward central and regional legislatures. In
1988 he undermined the power of the central party apparat and later
eliminated Article 6 of the Soviet Constitution, which had granted the
CPSU a monopoly.49 After the 1991 coup attempt, the party’s remaining
legitimacy rapidly drained away. The Soviet legislature suspended all
party activity, and two months later Yeltsin formally banned the Com-
munist Party in Russia.50 With the party’s collapse, the Soviet Union’s
main integrative institution disappeared, shattering accountability of
local leaders to the center.

Vertical accountability was also eroded by Gorbachev’s decision to
hold semicompetitive elections in 1990 to replace nomenklatura in fed-
eral and local legislatures, or Supreme Soviets. Though the nomenkla-

46 See Andrew Wilson’s discussion of nationalism in Ukraine in Wilson, The Ukrainians (New
Haven: Yale University Press, 2000); and Stephen Kotkin, Armageddon Averted: The Soviet Collapse,
1970–2000 (New York: Oxford University Press, 2001).

47 Jeffrey Hahn, “Introduction: Analyzing Parliamentary Development in Russia,” in Hahn, ed.,
Democratization in Russia: The Development of Legislative Institutions (Armonk, N.Y.: M. E. Sharpe,
1996).

48 Olga V. Kryshtanovskaia, “Transformation of the Old Nomenklatura into a New Russian Elite,”
Russian Social Science Review 37 ( July–August 1996).

49 Kotkin (fn. 46), 75–85.
50 Stephen White, Graeme Gill, and Darrell Slider, The Politics of Transition: Shaping of a Post-

Soviet Future (Cambridge: Cambridge University Press, 1993).

S E C E S S I O N I S M F R O M T H E B O T T O M U P 293

tura retained a majority of seats,51 independent deputies now served
alongside them in many republics, introducing ideas unthinkable just
a few years earlier. Competitive elections transformed rubber-stamp
soviets (councils) into fl edgling representative institutions whose legiti-
macy originated in the regions, rather than in Moscow.52

Furthermore, Gorbachev’s democratization policies allowed opposi-
tion groups to emerge and openly articulate heterodox ideas without
the expectation of serious state repression.53 Prodemocracy, environ-
mental, and ethnonationalist groups appeared. In some republics the
support base of nationalist groups remained confi ned to a small layer of
intelligentsia in the capital cities. But elsewhere, the nationalists’ popu-
larity grew, as teachers, students, and government offi cials were joined
by workers and party members. In Tatarstan, Tuva, Chechnya, Yakutia,
and Bashkortostan, opposition nationalist movements were developing
a mass following.

Republican leaders watched as new constituencies began seizing
the political initiative. They observed rising support for nationalism
by looking out their offi ce windows at mass demonstrations held on
central squares, by reading impassioned debates in the local press, and
by monitoring nationalist group activity. They watched as pillars of the
Soviet establishment became targets of organized campaigns against
corruption and mismanagement. The nationalist and democratic oppo-
sition in Bashkortostan, for example, led a grassroots campaign against
the republic’s party committee that led to its expulsion from power.54
Nationalist organizations formed national congresses in an attempt to
supplant republican Supreme Soviets. Though unsuccessful, certain de-
mands raised at the national congresses prompted Supreme Soviets to
respond. When, for example, the founding meeting of the Chechen
National Congress in 1990 passed a resolution on state sovereignty,
Chechnya’s Supreme Soviet did the same days later.

Most important, nationalist organizations sponsored mass demon-
strations, rallies, pickets, and an occasional hunger strike in front of

51 Only in the union republics of Moldavia, the Baltics, and Armenia, as well as in parts of Ukraine,
did independent candidates win a signifi cant number of seats in Supreme Soviet elections.

52 For example, in Tatarstan, nationalists won one-third of the seats in the Supreme Soviet and
came to control approximately 120 out of 250 votes by winning over independent deputies. See I. V.
Terent’eva, R.Iu. Beliakov, and M.F. Safarov, Politicheskie partii i dvizhenie Respubliki Tatarstan (Politi-
cal parties and movements in Tatarstan) (Kazan, 1999); and Gorenburg (fn. 16), 135. See also Gavin
Helf and Jeffrey Hahn, “Old Dogs and New Tricks: Party Elites in the Russian Regional Elections of
1990,” Slavic Review 51 (Fall 1992).

53 See Herrera’s discussion (fn. 8) on breaking the Soviet doxa (pp. 98–142).
54 Subsequently, several members of the Bashkir nationalist movement joined the republic’s interim

leadership. RFE/RL Research Report 2, no. 8 (February 23, 1990); and “Plenary Sessions Held,” Current
Digest of the Soviet Press 42, no. 6 (1990).

294 W O R L D P O L I T I C S

government buildings where they could attract maximum attention.
Tatarstan, for example, was home to 142 nationalist demonstrations
with thousands of participants from 1987 to 1994. The most radical
rallies demanding independence continued for days in October 1991.
When demonstrators heard that the Supreme Soviet had voted against
a declaration of independence, they rushed the building. As one parlia-
mentary deputy remembers:

I will never forget how the crowds, the multi-thousand person crowds sur-
rounded our parliament building on Freedom Square from morning till night
and how they idolized Fauzia Bairamova [a radical nationalist leader] who was
advocating secession from Russia. It was quite frightening. I was afraid that
such events might scandalously develop into a civil war. I remember . . . that
when . . . we [the deputies] exited the parliament, the militia was holding back
the crowds with much diffi culty . . . and they were throwing pieces of broken
glass and nails into us.55

These acts stimulated public debate and won massive media attention,
while demonstrators risking arrest, job loss, and physical injury signaled
their strong commitment to their cause. All of this increased the power
and resources nationalists wielded within the political arena.

For nomenklatura leaders unaccustomed to considering popular
opinion, a misstep was potentially fatal. The president of Tatarstan,
Mintimir Shaimiev, learned this lesson after he backed the attempted
coup against Gorbachev, suppressing information and arresting people
demonstrating against the leaders of the coup. When the coup failed,
opposition groups began a grassroots campaign to impeach Shaimiev.
He was eventually absolved when nationalist groups decided that, as an
ethnic Tatar, Shaimiev would ultimately back their interests. Shaimiev
then announced support for republican sovereignty—a position the
nationalists had been advocating since glasnost began.56 The fact that
Shaimiev, a good communist apparatchik, supported saving the So-
viet Union one moment and demanded state sovereignty the next sug-
gests the impressive political opportunism that produced secessionism
in Russia’s republics.

Republican leaders could not even necessarily count on enterprise
directors to continue to support the status quo. Directors of factories

55 Jeffrey Kahn, interview with Boris Leonidovich Zheleznov, professor of law, Kazan State Uni-
versity, Kazan, June 6, 1997. Similarly, the Union of Bashkir Youth marched on Bashkortostan’s tele-
vision station and went on air to denounce the Supreme Soviet’s vote to delay presidential elections.
RFE/RL Research Report 3, no. 47 (November 22, 1991).

56 See Elise Giuliano, “Who Determines the Self in the Politics of Self-Determination? Ethnicity and
Preference Formation in Tatarstan’s Nationalist Mobilization,” Comparative Politics 32 (April 2000).

S E C E S S I O N I S M F R O M T H E B O T T O M U P 295

and mines had fi nancial incentives to support the nationalist goal of
republican sovereignty, realizing that they could exploit central state
weakness to increase ownership of state property under their manage-
ment. Some directors had already begun to pursue personal profi t by
forming cooperatives that marketed the products of the state-owned
enterprises they managed.57 In general, perestroika benefi ted enterprise
directors by releasing them from following the economic plan and by
reducing monitoring by ministries in Moscow. The increase in local
control offered by republican sovereignty could further enlarge these
benefi ts. In the event, enterprise directors of lucrative large enterprises
in the nationalist republics did profi t fi nancially from the increased au-
tonomy their republics won from Moscow.58 This autonomy was the
direct result of campaigns for sovereignty initiated by their republics.

In addition, the demonstration effect of nationalism in certain Union
republics served as warnings to leaders in Russia’s republics. Together
with their populations, they observed how radicalizing nationalist crowds
forced out even popularly elected presidents Gamsukhurdia in Georgia
and Mutalibov in Azerbaijan.59 Thus, while republican leaders used the
threat of ethnic unrest during negotiations with Moscow, they did not
foment nationalism simply to strengthen their bargaining position. The
risk inherent in such behavior was clear. Popular opinion—instantiated
in nationalist rallies, intense public debate, and occasional violence—
could turn against them. Like politicians everywhere, republican lead-
ers wanted to satisfy constituents and keep their jobs. Their demands
for sovereignty from Moscow were motivated by self-interest.

An alternative hypothesis might argue that the key variable infl u-
encing secessionism was whether republican incumbents repressed
or co-opted nationalist groups soon after they formed, denying them
the ability to win popular support and pressure local leaders. Yet com-
paring the republics, we observe no correlation between repression/
co-optation and weak nationalist movements, or between nonrepres-
sion/non-co-optation and popular nationalist movements. Party leader-
ships in most republics were changing as a result of personnel reforms
in 1989, Supreme Soviet elections in 1990, and the anti-Gorbachev

57 This began after Gorbachev passed the 1987 Law on Cooperatives. Eric Hanley, Natasha Yer-
shova, and Richard Anderson, “Russia—Old Wine in a New Bottle? The Circulation and Reproduc-
tion of Russian Elites, 1983–1993,” Theory and Society 24 (October 1995).

58 As did former nomenklatura political leaders. For example, the oil company Tatneft underwent
little restructuring and remained controlled by Tatarstan’s Soviet-era managers. See Leo McCann,
“Embeddedness, Markets and the State: Observations from Tatarstan,” in McCann, ed., Russian
Transformations (New York: RoutledgeCurzon, 2004); and Jozsef Borocz and Akos Rona-Tas, “Small
Leap Forward: Emergence of New Economic Elites,” Theory and Society 24 (October 1995).

59 White, Gill, and Slider (fn. 50), 103.

296 W O R L D P O L I T I C S

coup in 1991 and were therefore either unable to repress or uninter-
ested in doing so. For example, Chuvashia’s leadership was under fi re
from the democratic opposition for corruption, failed to control the
1990 parliamentary elections, and was ultimately deposed for support-
ing the coup.60 This gave Chuvashia’s nationalists an opening to attract
support, though they nonetheless failed to do so. Also conservatives did
not co-opt or repress nationalist movements in republics where they
retained power. In Mari El, a conservative Supreme Soviet chairman
replaced a conservative Obkom leader, yet the nationalist group Mari
Ushem was able to fi eld a presidential candidate. Despite this, Mari
Ushem attracted minimal popular support.61 Finally, in Tatarstan, na-
tionalist groups won support even when authorities used repressive tac-
tics early on to dilute their infl uence. In short, republican leadership
was contested during this time and executives did not control repub-
lican politics.62 By the mid-1990s, however, when popular support for
nationalism had dwindled, republican presidents were able to disregard
the weakened opposition and consolidate power.

CAMPAIGNS FOR SOVEREIGNTY IN RUSSIA’S REPUBLICS
Popular support for nationalism infl uenced republican secessionism both
directly, by electing nationalist candidates to republican legislatures, and
indirectly, by pressuring incumbent leaders concerned with self-preser-
vation. I provide two types of evidence of the effect of mass nationalism
on republican secession. First, I show that both the number of ethnic
demonstrations and the instances of ethnic violence in the republics are
correlated with the level or index of republican secessionism (Table 1).
Four fi gures below depict the relationship between these variables. Sec-
ond, case-study evidence from Tuva and Yakutia shows how mass na-
tionalism increased those republics’ secessionist demands on Moscow,
while evidence from Mari El indicates that low mass nationalism kept
secessionism to a minimum there.

Participation in ethnic demonstrations captures support for nation-
alism because it suggests peoples’ willingness to engage in risky behav-
ior to express opposition to current state authority. Also, an individual’s
decision to step onto the streets represents a public assertion of com-
munity and national identity.63 In Figure 3, we observe a relationship

60 Gorenburg (fn. 16), 62–63, 72.
61 Timur Muzaev, Etnicheskii separatizm v Rossii (Ethnic separatism in Russia) (Moscow: Pan-

orama Publishers, 1999), 159–63.
62 White, Gill, and Slider (fn. 50), 101.
63 See Mark Beissinger’s discussion of the meaning of nationalist event; Beissinger, Nationalist Mo-

bilization and the Collapse of the Soviet State (New York: Cambridge University Press, 2002), 18–28.

S E C E S S I O N I S M F R O M T H E B O T T O M U P 297

between the number of demonstrations and the republics with the
highest secessionism scores. These two variables are signifi cantly cor-
related (.661; p≤.005).64 For example, the highly secessionist Tatarstan
and Checheno-Ingushetia had 77 and 58 demonstrations, respectively,
while the minimally secessionist republics of Mordova and Mari El had
none. However, in other secessionist republics, there were few demon-
strations: Bashkortostan (10), Tuva (4), and Yakutia (3). This could be
explained by the fact that some nationalist groups chose the strategy

64 The variables of ethnic demonstrations and ethnic violence are from Mark Beissinger’s data set
of mobilization events in Soviet Union. I am grateful to him for providing these data. Beissinger de-
fi nes an ethnic demonstration as a “voluntary gathering of persons with the purpose of engaging in a
collective display of sentiment for or against public policies.” All demonstrations had a minimum of
one hundred participants. Beissinger defi nes a mass violent event as “a mass political action whose pri-
mary purpose was to infl ict violence, either in the form of an attack on people or on property.” Violent
events had a minimum of fi fteen participants. The data set does not include hunger strikes or general
strikes. See Codebook, “Non-Violent Demonstrations and Mass Violent Events in the Former USSR,
1987–1992,” 4, 6. For more on the procedures utilized in collecting the data, see Mark R. Beiss-
inger, “Event Analysis in Transitional Societies: Protest Mobilization in the Former Soviet Union,”
in Dieter Rucht, Ruud Koopmans, and Friedhelm Neidhardt, eds., Acts of Dissent: New Developments
in the Study of Protest (Berlin: Sigma Press, 1998); Beissinger (fn. 63); and http://www.polisci.wisc.
edu/~beissinger/research.htm.

298 W O R L D P O L I T I C S

FIGURE 3
NUMBER OF ETHNIC DEMONSTRATIONS IN RUSSIA’S REPUBLICS VERSUS

INDEX OF SECESSIONISM

N
um

be
r

of
E

th
ni

c
D

em
on

st
ra

ti
on

s
0

2
0

40

6

0

8

0

0 2 4 6 8 10 12
Index of Secessionism

of sponsoring candidates in local elections instead of holding demon-
strations. Moreover, the number of demonstrations in some republics
may be underrepresented because the data set used here concludes in
the year 1992, whereas republican campaigns for sovereignty continued
until 1994.

The late Soviet and early post-Soviet years generated much popular
protest by many groups on a variety of issues. Examining ethnic dem-
onstrations as a percentage of total demonstrations, we see in Figure 4
that this variable is also correlated with republican secessionism (.518;
p≤.05.). In the highly secessionist republics of Tatarstan, Tuva, and
Checheno-Ingushetia, a majority of demonstrations (between 60 per-
cent and 100 percent) concerned ethnic issues. In the secessionist repub-
lics of Yakutia and Bashkortostan between 40 percent and 50 percent
of demonstrations concerned ethnic issues. Note that the three North
Caucasian republics of Dagestan, Kabardino-Balkaria, and North Os-
setia have low secessionism scores yet a high number of ethnic demon-
strations. This can be explained by the fact that most demonstrations in
these republics concerned issues other than ethnonational sovereignty.

S E C E S S I O N I S M F R O M T H E B O T T O M U P 299

FIGURE 4
NUMBER OF ETHNIC DEMONSTRATIONS AS A PERCENTAGE OF TOTAL

DEMONSTRATIONS IN RUSSIA’S REPUBLICS VERSUS INDEX OF SECESSIONISM

N
um

be
r
of
E
th
ni
c
D
em
on
st
ra
ti
on

s,
%

o
f T

ot
al

0

2

0

40

6
0

8

0

0 2 4 6 8 10 12
Index of Secession

For example, the repatriation of ethnic groups deported by Stalin and
the wars between Georgia and the republics of Abkhazia and South
Ossetia were major issues in these republics.65 That is, ethnic dem-
onstrations there revolved around intraethnic confl icts and displays of
sympathy with neighboring Abkhazia and South Ossetia and did not
develop into secessionism.

Several republics experienced ethnic violence during the years 1987–
92. This violence was related to campaigns for sovereignty in some,
though not all, republics. As shown in Figure 5, the largest number of
violent ethnic events occurred in the secessionist republics of Chech-
eno-Ingushetia (21), Tatarstan (6), Tuva (7), and Yakutia (2). In con-
trast, the low-secession republics witnessed little to no ethnic violence
during this period. North Ossetia is again an outlier because of the vio-
lent confl ict that took place between Ingush and Ossetians in fall 1992;
it was not related to a campaign for sovereignty.66 Thus, mass violent events
and secessionism were statistically correlated but less strongly so than the
variables of ethnic demonstrations and secessionism (.438; p≤.10).

If we consider ethnic mass violent events as a percentage of all mass
violent events that occurred, we fi nd a very strong and statistically sig-
nifi cant correlation between this variable and republican secessionism
(.768; p≤.0005). Between 84 percent and 100 percent of mass vio-
lent events concerned ethnic violence in the secessionist republics of
Tatarstan, Tuva, Checheno-Ingushetia, and Yakutia. (See Figure 6.)
Again, Dagestan and North Ossetia are outliers.

Although demonstrations and ethnic violence are suggestive, they
are not comprehensive indicators of popular support for nationalism.
Case-study evidence more accurately captures how mass nationalism
infl uenced republican campaigns for sovereignty at critical junctures.
Below, I compare the republics of Tuva, Yakutia, and Mari-El, which
differ in terms of popular support for nationalism. Tuva and Yakutia
are representative of republics where mass nationalism pressured incum-
bent leaders into adopting nationalist policies and making secessionist
demands. Mari-El, conversely, is typical of those republics where low

65 Concerning repatriation issues, Akkintsy clashed with neighboring ethnic groups in Dages-
tan, Balkars challenged Kabardins in Kabardino-Balkaria, and Ingush made claims in the Prigorodnii
region of North Ossetia. In terms of the regional armed confl icts, North Ossetia supported South
Ossetia against Georgia by taking Ossetian refugees and sending aid and volunteer fi ghters, while
Kabardino-Balkaria took the side of Abkhazia against Georgia. See Jane Omrod, “The North Cauca-
sus: Confederation in Confl ict,” in Ian Bremmer and Ray Taras, eds., New States, New Politics (Cam-
bridge: Cambridge University Press, 1997).

66 Tomila Lankina discusses ethnic confl ict and mobilization in North Ossetia in Governing the Locals:
Local Self-Government and Ethnic Mobilization in Russia (NewYork: Rowman and Littlefi eld, 2004).

300 W O R L D P O L I T I C S

levels of mass nationalism allowed incumbents to ignore opposition na-
tionalist movements and remain in offi ce.67 In addition, the compari-
son of Tuva and Yakutia undermines the wealth hypothesis because
the republics differ in terms of republican wealth but exhibit a com-
mon outcome of secessionism. The fact that separatism emerged in the
territorially tiny, economically underdeveloped, and resource-poor re-
public of Tuva and also in the enormous and resource-rich republic of
Yakutia suggests that the variable of wealth provides scant causal lever-
age. Finally, the republics of Tuva, Yakutia, and Mari-El are neglected
in most studies of nationalism in Russia, which generally examine the
high-profi le republics of Chechnya and Tatarstan and ignore republics
with low nationalism.

67 I use the diverse-case method and the typical-case method to select cases, according to John
Gerring’s typology in Gerring, Case Study Research: Principles and Practice (New York: Cambridge
University Press, 2007).

S E C E S S I O N I S M F R O M T H E B O T T O M U P 301

FIGURE 5
NUMBER OF ETHNIC MASS VIOLENT EVENTS IN RUSSIA’S REPUBLICS VERSUS

INDEX OF SECESSIONISMa

aDue to space constraints, the names of several republics are abbreviated: M-E = Mari-
El, Udm=Udmurtia, B=Buryatia, C=Chuvashia, KB=Kabardino-Balkaria, K=Komi, Kal=
Kalmykia, Kar=Karelia.

N

um
be

r
of

M
as

s
V

io
le

nt
E

ve
nt

s
0

5
1

0
15

20

25

0 2 4 6 8 10 12
Index of Secessionism

THE REPUBLIC OF TUVA
Tuva—a tiny Siberian republic of 309,000 people in 1989, with an
economy centered on traditional livestock herding of sheep, cattle,
and horses—was highly secessionist and offers strong evidence that
popular support for nationalism infl uenced republican campaigns for
sovereignty.68 First, several members of nationalist organizations were
elected to Tuva’s Supreme Soviet in 1990 and 1993 and the leader of
the nationalist movement served in the highest post in the republic—
chairman of the Supreme Soviet. Second, the republic experienced very
high levels of ethnic violence in the early 1990s. Third, support for na-
tionalism infl uenced the substance and adoption of Tuva’s constitution,
as well as the vote against the constitution of the Russian Federation.

68 Moscow appropriated what little gold, uranium, cobalt and coal Tuva, one of Russia’s poorest
republics, did contain. Zoia Anaibin, “Ethnic Relations in Tuva,” in Marjorie Balzer, ed., Culture In-
carnate (Armonk, N.Y.: M. E. Sharpe, 1995).

302 W O R L D P O L I T I C S

FIGURE 6
NUMBER OF ETHNIC MASS VIOLENT EVENTS AS A PERCENTAGE OF TOTAL
VIOLENT EVENTS IN RUSSIA’S REPUBLICS VERSUS INDEX OF SECESSIONISMa

aDue to space constraints, the names of several republics are abbreviated: M-E = Mari-
El, Udm=Udmurtia, B=Buryatia, C=Chuvashia, KB=Kabardino-Balkaria, K=Komi, Kal=
Kalmykia, Kar=Karelia.
N
um
be
r

of
M

as
s

V
io

le
nt

E
ve

nt
s

0

20

4
0

60

80

0 2 4 6 8 10 12
Index of Secessionism

The nationalist movement in Tuva began in 1989 with the formation
of the Popular Front of Tuva. The Front’s leader, Kaadyr-ool Bicheldei,
grew increasingly popular and was elected to represent Tuva in the
RSFSR Congress of Peoples’ deputies.69 At the same time, ten mem-
bers of the Popular Front won seats in Tuva’s Supreme Soviet in the fi rst
competitive elections of 1990. Support for Bicheldei rose further when
Tuva’s conservative Supreme Soviet chairman, D. Ondar, backed the
plotters during the 1991 coup against Gorbachev. A series of protest
demonstrations and hunger strikes followed, leading to Ondar’s ouster
and replacement by Bicheldei. With Bicheldei’s appointment, Tuva be-
came the only republic in which a leader of the nationalist movement
held the top post of Supreme Soviet chairman.70

In the summer of 1990 an intense spate of Russian-Tuvan violence
erupted.71 Following a fi ght between young Tuvan and Russian “public
order volunteers” at a disco in an industrial town, Tuvan gangs fanned
out through the town destroying property, intimidating the popula-
tion, and clashing with police. The violence expanded to other cities.
In the capital of Kyzyl, members of Tuvan gangs shot several Russians,
reportedly for not knowing how to speak Tuvan. In another city, Tu-
vans burned and raided Russians’ homes, leaving notes urging them to
get out of town.72 Communist Party fi rst secretary Gregorii Shirshin,
a Brezhnev-era appointee, tried to discredit the Popular Front of Tuva
by blaming the group for inciting anti-Russian pogroms. Bicheldei ve-
hemently protested the allegations, appealed to the KGB to clear the
group’s name, but decided to formally dissolve his organization to allay
concerns.

Two new nationalist organizations evolved out of the dismantled
Popular Front: the moderate National Party of Sovereign Tuva (NPST)
and the radical Peoples’ Front Khostug Tuva (Free Tuva). Both groups
were composed exclusively of ethnic Tuvans and shared the goal of
strengthening republican sovereignty. Khostug Tuva, whose members

69 Ann Sheehy, “Russians the Target of Interethnic Violence in Tuva,” RFE/RL Research Report 2, no.
37 (September 14, 1990).

70 According to one Russian analyst, Tuva’s Popular Front represented a serious force in republican
politics due to its mass popular support. Muzaev (fn. 61), 186–89.

71A question exists as to whether the violence was Tuvan on Tuvan rather than interethnic. It
was nevertheless perceived by the press and the general population as ethnic violence. Toomas Ala-
talu, “Tuva: A State Reawakens,” Soviet Studies 44, no. 5 (1992). See also Marjorie Mandelstam Bal-
zer, “From Ethnicity to Nationalism: Turmoil in the Russian Mini-Empire,” in James R. Millar and
Sharon L. Wolchik, eds., The Social Legacy of Communism (Cambridge: Cambridge University Press,
1994).

72 Overall, by the mid-1990s, approximately ten thousand Russians, including highly skilled engi-
neers, teachers, and doctors, were estimated to have migrated out of Tuva. Stefan Sullivan, “Inter-
ethnic Relations in post-Soviet Tuva,” Ethnic and Racial Studies 18, no. 1 (1995).

S E C E S S I O N I S M F R O M T H E B O T T O M U P 303

were young, unemployed migrants from the republic’s rural regions,
began to organize rallies in 1992. The NPST, made up mainly of profes-
sional politicians and members of the intelligentsia, organized as a politi-
cal party to defend Tuvan sovereignty. The NPST established local organs
throughout Tuva and actively campaigned in the 1993 elections to the
now renamed Supreme Soviet—the Supreme Khural. The party’s grow-
ing authority among Tuvans translated into electoral success: it won 62.9
percent of the popular vote and three out of fi ve seats in the Khural re-
served for political parties. In the same elections, a leading NPST member
and scientist at the Tuvan branch of the Russian Academy of Sciences,
Kara-Kyz Dongakovny Arakchaa, won a seat in the Russian Duma.

Popular support for nationalism exerted the greatest pressure on re-
publican policy during the crafting and adoption of Tuva’s constitu-
tion in 1993. Two issues defi ned public debate throughout the early
1990s: the question of Tuva’s new constitution and the republic’s right
to secede from Russia. Following a popular referendum, both NPST and
Khostug Tuva actively lobbied for a constitutional amendment on se-
cession.73 The issue of secession was widely debated in the media and
among the general population in “kitchen conversations.” Many people
believed that Tuva met the three conditions articulated by the Russian
sociologist and human rights advocate Galina Starovoitova as necessary
for secession: a titular majority, an international border (with Mongo-
lia), and past independent statehood.74 According to a local (Russian)
analyst whose husband served in Tuva’s parliament during the debate,
the drafting of the constitution elicited high concern among the popu-
lation, with most people coming to agree by 1992 that the constitution
should include a right to secede.

Parliamentary deputies in the Supreme Khural heeded popular opin-
ion and voted in favor of an amendment on secession. The amendment
was to serve as a warning to Moscow that Tuva would consider seces-
sion if its sovereignty were infringed upon. Because nationalist groups
had been lobbying for the right to secede since their inception, they
considered the amendment’s inclusion to be a victory for their move-
ment.75 In addition, another amendment backed by the nationalists—
a ban on private landownership—was included in the constitution, as
was the statement that republican law took primacy over federal law.

73 Khostug Tuva went a step further, calling for the republic to hold an immediate referendum on
secession from Russia; Drobizheva (fn. 18), 125.

74 Tuvans formed 65 percent and Russians 32 percent of the population in 1989. Tuva was nomi-
nally independent from 1921 to 1944, when it was incorporated into the USSR. Before 1921 it held
the status of a Russian protectorate. Balzer (fn. 71), 124.

75 Anaibin (fn. 68), 104, 110.

304 W O R L D P O L I T I C S

The NPST also devoted its energy to opposing Russia’s federal con-
stitution. The group sponsored demonstrations against the adoption
of the constitution during the fall of 1993. Its campaign was a success;
when both the federal and Tuvan constitutions were put to a popular
vote on the same day, a majority of Tuva’s population voted for Tuva’s
constitution (53.9 percent) but against the federal constitution. Only
29.8 percent of Tuvans voted to approve the federal constitution.76 The
results of these votes indicate that Tuva’s population supported the na-
tionalist policies of an amendment on secession, a ban on private land-
ownership, and the precedence of Tuvan law over federal law.77

Despite the infl uence of the Popular Front in republican politics, by
the mid-1990s popular support for nationalism in Tuva was declin-
ing.78 This, together with Yeltsin’s consolidation of central state control,
began to shift political power away from the parliament led by Bich-
ildei and toward the executive branch. Tuva’s new president, Shegir-
Ool Orzhak, co-opted nationalist leaders by offering them government
positions. Nevertheless, during Tuva’s autonomy campaign in the early
1990s, the nationalist movement attracted extensive levels of support
among the general population and was able to signifi cantly infl uence
republican policy.

THE REPUBLIC OF YAKUTIA
Yakutia was another of Russia’s secessionist republics, demanding the
right from Moscow to control its economy, including the sale of nat-
ural resources located on its territory. According to the conventional
wisdom, the existence of diamonds, gold, and coal motivated Yakutia’s
leaders to make secessionist demands.79 I argue, however, that the key
determinant of separatism was the presence of popular support for the
Yakut nationalist movement that was able to exert infl uence on repub-
lican policy. Evidence consists of, fi rst, the election of nationalist candi-
dates to republican legislatures; second, a moderate level of mass activ-
ity, including demonstrations, strikes, and instances of ethnic violence;

76 The popular vote took place on December 12, 1993. Tuva’s Supreme Khural had voted to adopt
Tuva’s constitution earlier that fall, in October, while Yeltsin was busy bombing the federal parliament.
A. Kuzhuget and M. Tatarintseva, Etnopoliticheskaia situatsiia v Respublike Tyva, Research Mono-
graph, no. 74 (Moscow: Institute of Ethnography and Anthropology, Russian Academy of Sciences,
1994), 9.

77 Ann Sheehy, “Tuva Adopts New Constitution,” RFE/RL Daily Report no. 203 (October 26, 1993), 2.
78 I address the decline of mass nationalism in my book manuscript, in Giuliano, “Why Secession

Fails: The Rise and Fall of Ethnic Nationalism in Russia.”
79 See, for example, Daniel R. Kempton, “The Republic of Sakha (Yakutia): The Evolution of Cen-

tre-Periphery Relations in the Russian Federation,” Europe-Asia Studies 48 ( June 1996).

S E C E S S I O N I S M F R O M T H E B O T T O M U P 305

and third, popular pressure on the republic’s Supreme Soviet surround-
ing the adoption of Yakutia’s constitution.

The two main nationalist groups in Yakutia were the moderate um-
brella organization Sakha Omuk (the Sakha People) and the more rad-
ical Sakha Keskile (Sakha Perspective). Sakha Keskile advocated union
republic status for Yakutia and actively supported republican sover-
eignty. Its founder, the linguist Lazar Afanas’ev, pithily summarized
the group’s stance as “our sharp politics against those in power.” The
group’s support for Boris Yeltsin during the 1991 coup provoked Ya-
kutia’s Communist Party to fi re members of Sakha Keskile from their
jobs in provincial republican cities in order to slow the group’s grow-
ing infl uence.80 Sakha Omuk, the moderate group, advocated cultural
rebirth of the Sakha nation and greater Yakut representation in the top
tiers of republican administration and enterprise management; it also
demanded that Yakutia keep a greater share of the income from its
natural resources.

The nationalist opposition began to win popular support. Several
Sakha Omuk candidates were elected to local soviets throughout Ya-
kutia and to the Supreme Soviet in 1990. Though nomenklatura con-
tinued to dominate, Yakut nationalists allied with democrats in the Su-
preme Soviet to declare sovereignty in 1990.81 Sakha Omuk lobbied
for the election of Mikhail Nikolaev as the republic’s fi rst president.
Nikolaev, a product of the Soviet establishment, had just been elected
chairman of Yakutia’s reconstituted Supreme Soviet in 1990. When the
post of republican president was created one year later, Nikolaev won
a majority (76.7 percent) of the popular vote. The nationalists backed
Nikolaev because he supported their interest in greater economic sov-
ereignty and because he was an ethnic Yakut who fulfi lled the require-
ment that the president be able to speak Yakut.82

In terms of mass activity, there were several isolated instances of in-
terethnic violence in Yakutia. Marjorie Mandelstam Balzer notes that
the everyday ethnic tension that existed between Yakuts and Russians
erupted into violence at several points in the 1980s and 1990s.83 Also,

80 Marjorie Mandelstam Balzer, “A State within a State: The Sakha Republic (Yakutia),” in Stephen
Kotkin and David Wolff, eds., Rediscovering Russia in Asia (Armonk, N.Y.: M. E. Sharpe, 1995).

81 After declaring sovereignty, Yakutia changed its name to The Sakha Republic (Yakutia).
82 F. M. Zykov, Etnopoliticheskaia situatsiia v Respublike Sakha do i posle vyborov 12 dekabria 1993 g.,

Research Monograph, no. 71 (Moscow: Institute of Ethnography and Anthropology, Russian Acad-
emy of Sciences, 1994). Khazanov (fn. 12) writes that Nikolaev “liked to point out [during negotia-
tions with Yeltsin] that he acted under heavy pressure from Sakha Omuk and that some decisions of
Yakutia’s parliament were made against his will” (p. 183).

83 Marjorie Mandelstam Balzer and Uliana Alekseevna Vinokurova, “Nationalism, Interethnic Rela-
tions and Federalism: The Case of the Sakha Republic (Yakutia),” Europe-Asia Studies 48 ( January 1996).

306 W O R L D P O L I T I C S

Yakutia’s coal, diamond, and gold miners went on strike during the
fi nal years of the Soviet Union. For example, in 1990, in the diamond-
mining town of Mirny, where most miners were recent ethnic Russian
migrants, a short-lived Russian movement developed, advocating se-
cession from Yakutia and the creation of a “Lena Republic.” A rela-
tively low number of demonstrations occurred in Yakutia as the nation-
alist movement chose not to employ the strategy of mass protest rallies.
However, a demonstration held in 1992 indicates how a strong display
of popular opinion infl uenced policy at a critical juncture in republican
politics: the vote on Yakutia’s constitution.

By spring 1992 Yakutia’s parliament and population had already been
considering for some time whether to adopt a constitution—an act that
would be the fi rst of its kind among Russia’s republics. The general
population and Yakuts in particular actively participated in the debate
surrounding its adoption. There were two main issues of contention:
whether the constitution should be passed before or after the Russian
constitution and whether it should include a right to secede. As the Su-
preme Soviet opened its session in April, expectations had risen among
the population that it would be adopted. When it became apparent
that the parliament was leaning toward allowing Moscow to fi rst hold
its vote on the federal constitution, nationalist groups organized a mass
demonstration in front of the parliament building calling for the con-
stitution’s immediate passage with an amendment guaranteeing a right
to secede. The rally was well attended, both by Yakut intelligentsia and
by youth from the nationalist student organization Eder Saas. Protest-
ers at the rally threatened to replace the Supreme Soviet chairman with
the vice chairman if the constitutional vote was not held.84

The parliament responded to this pressure by voting for immediate
adoption of the constitution and for inclusion of the controversial clause
on secession. Sakha Keskeli’s infl uence among the population prompted
the Supreme Soviet to meet the group’s call for a constitutional right
to secede. Another of Sakha Keskeli’s demands, however—that the Ya-
kut nation be labeled the “bearers of state sovereignty”—was rejected.
Nevertheless, overall, the passage of the constitution represented an
enormously important symbolic event in the republic, following a year
and a half of tense national debate. Yakutia’s constitution asserted the
supremacy of republican law over federal law and provided for the right
to secede. The nationalists considered its adoption to be their victory
and organized a national (vsyenarodnaia) public holiday in the central

84 Drobizheva (fn. 18) notes that this threat indicated a very radical mood among Yakuts since it
would have meant replacing a Yakut with an ethnic Russian (pp. 135–52).

S E C E S S I O N I S M F R O M T H E B O T T O M U P 307

square of the city where people celebrated the long-anticipated constitu-
tion.85 By the mid-1990s, however, the nationalist movement’s popular-
ity had declined. But the considerable popular support it had enjoyed in
the early 1990s allowed it to infl uence key republican policy.

THE REPUBLIC OF MARI-EL
Mari-El, with a population almost evenly split between Russians and
Maris, was among the least secessionist of Russia’s republics. Its leaders
advanced few initiatives following a 1990 declaration of sovereignty.
For example, the republic’s Supreme Soviet failed to pass a draft lan-
guage law written in the early 1990s and did not adopt a new constitu-
tion until 1997.86 Mari-El had two nationalist organizations that found
little popular support as indicated by few mass demonstrations, no eth-
nic confl ict, and the nationalists’ meager electoral success and lack of
infl uence on republican policy.

The radical nationalist group, Kugeze Mlande (Land of our Ances-
tors) advocated secession from Russia and exclusivist laws on migra-
tion. Kugeze Mlande’s provocative statements and rallies brought the
group attention, but it was unpopular and never succeeded in taking
control of the nationalist movement.87 The moderate nationalist orga-
nization, Mari Ushem (Mari Union), as in other republics, supported
republican sovereignty, state status for the Mari language, cultural re-
vival, and the end of Russifi cation. Mari Ushem, however, chose not
to become an opposition movement and from its inception cooperated
with local leaders.

According to a local expert on republican politics, Mari Ushem’s in-
fl uence on public opinion was insignifi cant, despite the fact that most
people, especially ethnic Mari, were well informed about the group.88
First, Mari Ushem backed the losing candidate in the republic’s fi rst
presidential election.89 Second, results from the 1993 elections to the
federal Duma and to Mari-El’s Supreme Soviet confi rm the group’s
lack of infl uence: only one Mari Ushem member won a Duma seat,
and none of its fi ve candidates were elected to the Supreme Soviet. The
radical group Kugeze Mlande failed to win any seats at all.

85 Ibid., 137; see also Muzaev (fn. 61), 206–10.
86 Mari-El passed a language law in 1995. Iurii P. Shabaev, “National Movements in the Eastern

Finnic Republics of Rossiia,” Anthropology and Archeology of Eurasia 37 (Fall 1998).
87 Muzaev (fn. 61), 159–63; V. D. Sharov, Etnopoliticheskaia situatsiia v Respublike Marii El, Re-

search Monograph, no. 63 (Moscow: Institute of Ethnography and Anthropology, Russian Academy
of Sciences, 1994), 5.

88 Sharov (fn. 87), 15.
89 Muzaev (fn. 61), 160–61.

308 W O R L D P O L I T I C S

Mari Ushem’s major initiative took place in spring of 1992, when it
organized a “First Congress of the Mari nation” in the capital Ioshkar-
Ola. The Congress passed resolutions calling for greater Mari repre-
sentation in government and identifying Maris as “a national minority
in their own republic.” Yet the Mari Congress remained moderate, op-
posing secession and supporting Yeltsin’s Federation Treaty, which was
to become part of the Russian Constitution. The following fall, the
republican administration began to co-opt the nationalists. It decided
to “legitimize the nationalist movement as the national representative
organization of the Mari nation” and fi nanced a second “First Congress
of the Mari Nation,” which passed resolutions identical to those passed
at Mari Ushem’s earlier congress. Although some government offi cials
were among the leaders of the Mari Congress, several other Supreme
Soviet deputies issued a formal protest to President Zotin stating that
the resolutions of the Congress contradicted existing representative or-
gans and laws. In response, Zotin had the resolutions of the Congress
published in the press along with a statement that they were mere rec-
ommendations without any legal status and that the government did
not share the group’s approach to solving problems.

After 1993 Mari Ushem began to concern itself mainly with language
and education issues.90 Ultimately, only a small portion of the republican
intelligentsia embraced the Mari nationalist movement. Both nationalist
groups signifi cantly decreased their activity as members stopped partici-
pating in elections and migrated to other voting blocs.91

SUMMARY OF CASE STUDIES
These case studies have shown that popular support for nationalism in
Tuva and Yakutia contributed to secessionism in those republics. In re-
publics that failed to make secessionist demands, such as Mari-El, pop-
ular nationalism was virtually absent. Political choices made by mass
populations, therefore, and the effect of those choices on the competi-
tion for power within republics can explain variation in secessionism
among Russia’s republics.

IV. CONCLUSION

This article has demonstrated that politics within subfederal regions
matters for the development of secessionism. It has shown how ethnic

90 Ronald Wixman and Allen Frank, “The Middle Volga: Exploring the Limits of Sovereignty,” in
Bremmer and Taras (fn. 65).

91 Muzaev (fn. 61), 162

S E C E S S I O N I S M F R O M T H E B O T T O M U P 309

federations undergoing democratization produce conditions that may
encourage regional separatism. Specifi cally, the transfer of political ac-
countability from center to region shifts the incentives of regional lead-
ers, forcing them to react to local constituencies in order to retain offi ce.
If these constituencies desire autonomy, regional leaders must respond,
making separatism not merely an opportunistic strategy but a neces-
sary one for the leaders’ own political survival. Democratization turns
administrative territories into electoral arenas. In Russia, the ethnic re-
publics became electoral arenas in which popular support for national-
ism pressured local leaders into making separatist demands. Thus, the
contest for political control of subfederal territories is a critical variable
driving regional secession.

These fi ndings have implications for other culturally plural societ-
ies concerned with preventing secession, such as Iraq and Afghanistan.
The fact of destabilizing campaigns for sovereignty in Russia in the
early 1990s implies that democratizing, multiethnic states should avoid
establishing subfederal territories that overlap with socially recognized
ethnic or religious communities. It is also the case that varying levels of
secessionism among Russia’s republics and the ultimate decline of na-
tionalism within the republics suggest that secessionism is not inevitable
but depends on the presence and degree of mass nationalism within
subfederal territories. And, this article has argued, mass nationalism
is neither a latent attribute of federal regions, nor a simple function of
natural resource endowments, nor something summoned into existence
by the manipulations of regional leaders. As such, attention should be
directed toward understanding the origins of popular nationalism in
ethnofederal states, as well as its role in the elite contest for power
within those states.

310 W O R L D P O L I T I C S

Still stressed with your coursework?
Get quality coursework help from an expert!